Chapter 20: COMPREHENSION


Passage 1




The history of human growth and development is at the same time the history of the terrible struggle of every new idea heralding the approach of a brighter dawn. In its tenacious hold on tradition, the Old has never hesitated to make use of the foulest and cruelest means to avoid the advent of the New, in whatever form or period the latter may have asserted itself. We need not retrace our steps into the distant past to realize the enormity of opposition, difficulties, and hardships placed in the path of every progressive idea. The rack, the thumbscrew, and the knout are still with us; so are the convict's garb and the social wrath, all conspiring against the spirit that is serenely marching on. Anarchism could not hope to escape the fate of all other ideas of innovation. Indeed, as the most revolutionary and uncompromising innovator, Anarchism must meet with the combined ignorance and venom of the world it aims to reconstruct.

The strange phenomenon of the opposition to Anarchism is that it brings to light the relation between so called intelligence and ignorance. And yet this is not so very strange when we consider the relativity of all things. The ignorant mass has in its favor that it makes no pretense of knowledge or tolerance. Acting, as it always does, by mere impulse, its reasons are like those of a child.

"Why?" "Because." Yet the opposition of the uneducated to Anarchism deserves the same consideration as that of the intelligent man. What, then, are the objections? First, Anarchism is impractical, though a beautiful ideal. Second, Anarchism stands for violence and destruction, hence it must be repudiated as vile and dangerous. Both the intelligent man and the ignorant mass judge not from a thorough knowledge of the subject, but either from hearsay or false interpretation.

A practical scheme, says Oscar Wilde, is either one already in existence, or a scheme that could be carried out under the existing conditions; but it is exactly the existing conditions that one objects to, and any scheme that could accept these conditions is wrong and foolish. The true criterion of the practical, therefore, is not whether the latter can keep intact the wrong or foolish; rather is it whether the scheme has vitality enough to leave the stagnant waters of the old, and build, as well as sustain, new life. In the light of this conception,Anarchism is indeed practical. More than any other idea, it is helping to do away with the wrong and foolish;more than any other idea, it is building and sustaining new life.

The emotions of the ignorant man are continuously kept at a pitch by the most blood-curdling stories about Anarchism. Not a thing too outrageous to be employed against this philosophy and its exponents. Therefore Anarchism represents to the unthinking what the proverbial bad man does to the child,-a black monster bent on swallowing everything; in short, destruction and violence.

Destruction and violence! How is the ordinary man to know that the most violent element in society is ignorance; that its power of destruction is the very thing Anarchism is combating? Nor is he aware that Anarchism, whose roots, as it were, are part of nature's forces, destroys, not healthful tissue, but parasitic growths that feed on the life's essence of society. It is merely clearing the soil from weeds and sagebrush, that it may eventually bear healthy fruit. Someone has said that it requires less mental effort to condemn than to think. The widespread mental indolence, so prevalent in society, proves this to be only too true. Rather than to go to the bottom of any given idea, to examine into its origin and meaning, most people will either condemn it altogether, or rely on some superficial or prejudicial definition of non-essentials


Q1)The passage is primarily concerned with

(1) identifying the contribution of the old ideologies to the present world
(2) exposing the dubious character of the intelligent men
(3) the novelty value of the idea of anarchism and its understanding
(4) revealing the untimely death of progressive ideas
(5) to express support to the basic idea of violence and destruction



Q2). Which of the following best describes the function of the fourth paragraph?

(1) To identify the flaws in the pragmatic and anarchistic approach
(2) To describe the complimentary relationship between the existing condition and anarchism
(3) To revert to the objections of the disbelievers of anarchism by magnifying its true pragmatic approach
(4) To refute Oscar Wilde's beliefs about the existing world and the practical scheme
(5) To identify the misgivings of the people toward Oscar Wilde's theory




Q3). By the author's statements, it can be inferred that she would be most likely to agree with which one of the following?

(1) The ignorant man's ignorance is not only employed by the lack of his own understanding but also by the efforts of those who want them to be oblivious.
(2) The acceptance of new ideas depends on that community of people which propagates those ideas
(3) Anarchism dichotomizes the intelligent from the ignorant and destroys those who have been thriving in society as parasites.
(4) The only pragmatic way is the present one which presents the correct order of things as proposed by our ancestors.
(5) That intelligence often gives way to Anarchism and is just a tool for the erudite to discuss in their living rooms.



Q4). The author is outraged by the judgement of anarchism from both the ignorant and intelligent men because

(1) their judgement is consistent with the exponents of anarchism
(2) their judgement is based on religion and social ethics
(3) their judgement is in opposition with the views of proponents of anarchism
(4) their judgement is based on unsubstantiated information and misconceptions
(5) their judgement is based on selfish motives


Q5) The tone of the author can be described as:


(1) adulatory (2) sympathetic (3) opinionated (4) equivocal (5)reverential


Solution to Passage 1


A - 1.


The passage begins by mentioning the historical struggle of new ideas but quickly moves onto the main subject of anarchism.




  • Option (1) sticks to the old ideologies which are not mentioned. In fact the author relates to the attitude of those who believe in the old ideology.


  • Option (2) is just one aspect of the entire view.


  • Option (4) suggests that progressive ideas die whereas the author does not conclude that in the passage.




  • Option (5) is too frivolous and it underlines the misconception that people have about Anarchism.


  • Option (3) conforms to the main subject of discussion and encompasses the misinterpretations related to understanding the idea. Hence,option (3) is correct.


A-2.


  • Option (1) is incorrect because the author relates to the practical scheme and the anarchistic approach.




  • Option (2) is incorrect because it suggests that the relation between existing condition and anarchism is complimentary whereas it is the existing condition that anarchism is disputing.


  • Option (4) is narrow in its approach as it only contains the essence of the first line of the fourth paragraph. The idea is not just to refute Oscar Wilde's belief but to make it work in favour of anarchism.


  • Option (5) is too broad and it deals with only one minor aspect of the passage.




  • Option (3) captures the true function of the paragraph by magnifying anarchism's practical side.


A- 3.


  • Option (2) is not discussed anywhere in the passage. The author does not talk about the difference of opinion among communities of people. Rather, it refers to the lack of acceptance of new ideas.


  • Option (3) is incorrect because the second paragraph clearly suggests the relation between the intelligent and ignorant men. It does not segregate the two.


  • Option (4) proposes that the old way of thinking is 'correct' which is clearly contradictory to the author's opinion.


  • Option (5) is beyond the scope of the argument.


  • Option (1) can be justified from the fifth paragraph -'the emotions of the ignorant are kept at pitch'.


A-4.


The last two lines of the third paragraph clearly explain the reason for author's discontentment with the ignorant and the intelligent men. Their judgement is based on 'hearsay and false interpretation'.


  • Options (1), (2) and (5) are not mentioned in the passage.




  • Option (3) is true, however, the author is outraged by their lack of reason in opposing anarchism and not merely that they oppose anarchism. Hence, option (4) is correct.



A-5.


Clearly, the author is opinionated as the passage unfolds her true feelings about the 'old' and how anarchism, like all other ideas, must 'meet the combined ignorance and venom of the world'.




  • Option (1) refers to praise or adulation which is not visible in the passage and option (2) refers to sympathy which cannot be interpreted from the passage.


  • Option (4) suggests doubtful or equivocal whereas the author seems to have a clear opinion on the issue.


  • (5) refers to regarding something as sacred, which is not the intention of the author in the passage. Hence, option (3) is correct

Reading comprehension - passage




The motive force that has carried the psychoanalytic movement to a voluminous wave of popular attention and created for it considerable following among those discontent with traditional methods and attitudes, is the frank direction of the psychological instruments of exploration to the insistent and intimate problems of human relations. However false or however true its conclusions, however weak or strong its arguments, however effective or defective or even pernicious its practice, its mission is broadly humanistic. Psychological enlightenment is presented as a program of salvation. By no other appeal could the service of psychology have become so glorified. The therapeutic promise of psychoanalysis came as the most novel, most ambitious, most releasing of the long procession of curative systems that mark the history of mental healing.

To the contemporary trends in psychology, psychoanalysis actually offered a rebuke, a challenge, a supplement, though it appeared to ignore them. With the practical purpose of applied psychology directed to human efficiency it had no direct relation and thus no quarrel. The solution of behaviorism, likewise bidding for popular approval by reducing adjustment to a program of conditioning, it inevitably found alien and irrelevant, as the behaviorist in reciprocity found psychoanalytic doctrine mystical, fantastic, assumptive, remote. Even to the cognate formulations of mental hygiene, as likewise in its contacts with related fields of psychology, psychoanalysis made no conciliatory advances. Towards psychiatry, its nearest of kin, it took an unfriendly position, quite too plainly implying a disdain for an unprogressive relative. These estrangements affected its relations throughout the domain of mind and its ills; but they came to head in the practice.

From the outset in the days of struggle, when it had but a sparse and scattered discipleship, to the present position of prominence, Freudianism went its own way, for the most part neglected by academic psychology. Of dreams, lapses and neuroses, orthodox psychology had little say. The second reason for the impression made by psychoanalysis when once launched against the tide of academic resistance was its recognition of depth psychology, so much closer to human motivation, so much more intimate and direct than the analysis of mental factors.



Most persons in trouble would be grateful for relief without critical examination of the theory behind the practice that helped them. Anyone at all acquainted with the ebb and flow of cures – cures that cure cures that fail – need not be told that the scientific basis of the system is often the least important factor. Many of these systems arise empirically within a practice, which by trial, seems to give results. This is not the case in psychoanalysis. Psychoanalysis belongs to the typical groups of therapies in which practice is entirely a derivative of theory. Here the pertinent psychological principle reads: “Create a belief in the theory, and the fact will create themselves”




Q.The distinctive feature of psychoanalysis is that


  1. it provided the laymen with a scientific basis to the theories of psychology
  2. it blasted the popular theory that the conscious mind could be aptly linked the tip of an iceberg.
  3. it provided effective means for the cure of mental disorders.
  4. it rendered existing trends in psychology defunct.

Ans.c




Q.The distinction between behaviorism and psychoanalysis that is heightened here is which of the following?


  1. Behaviorism is wide in scope; psychoanalysis more restricted.
  2. Behaviorism are more tolerant in their outlook; psychoanalysis more dogmatic.
  3. Behaviorism traces all action to conditioning by habit; psychoanalysis to the depths of the human mind
  4. Behaviorism are more circumspect and deliberate in their propagation of theory; psychoanalysis jump to conclusion impetuously

Ans.c


Q.The statement which is refuted by the passage is this:


  1. The popularity enjoyed by psychoanalysis is partly due to the disenchantment with traditional methods of psychology.
  2. Psychoanalysis wooed people dissatisfied with other branches of psychology to swell their ranks.
  3. Psychoanalysis were pioneers in the realm of analysis of the subconscious mind
  4. Psychoanalysis alienated allied branches of psychology.

Ans.b




Q.Create a belief in theory and


  1. belief will be created itself
  2. theory will be created itself.
  3. facts will be created themselves
  4. All of the above

Ans.c


Q.Psychoanalysis are of the opinion that


  1. methods of psychoanalysis must be in keeping with individual needs.
  2. inferences can be drawn empirically from repeated experiments with any given theory.
  3. theory leads to practice
  4. practice culminates into theory

Ans.c


Q.Freudian psychoanalysis was ignored by academic psychology because of which of the following?


  1. Its theories were not substantiated by practical evidence.
  2. It probed too deep into the human mind thereby divesting it of its legitimate privacy.
  3. It did not have a large following
  4. It was pre-occupied with unfamiliar concepts such as dreams and the subconscious mind.

Ans.d




Q.The only statement to receive support from the passage is which of the following?


  1. Psychoanalysis concentrated more on the theoretical remedies than their practical implementation
  2. Psychoanalysis broke the shackles of convention in its involvement with humanistic issues
  3. The attitude of psychoanalysis towards allied branches of psychology could at best be described as indifferent.
  4. Psychoanalysis dispelled the prevalent notion that dreams were repressed desires.

Ans.b


Q.The popularity enjoyed by the psychoanalytical movement may be directly attributed to


  1. dissatisfaction with existing methods of psychology
  2. its logical, coherent process of ratiocination.
  3. its novel unconventionality in both postulate and practice
  4. its concentration upon the humanistic aspect of psychological analysis.

Ans.d




Passage


It is undeniable that some very useful analogies can be drawn between the relational systems of computer mechanism and the relational systems of brain mechanism. The comparison does not depend upon any close resemblance between the actual mechanical links which occur in brains and computers; it depends on what the machines do. Further more, brains and computers can both be organized so as to solve problems. The mode of communication is very similar in both the cases, so much so that computers can now be designed to generate artificial human speech and even, by accident, to produce sequences of words which human beings recognize as poetry. The implication is not that machines are gradually assuming human forms, but that there is no sharp break of continuity between what is human, what is mechanical.


Q.From the passage, it is evident that the author thinks


  1. computers are now naturally programmed to produce poetry
  2. computers are likely to usurp the place of intellectual superiority accorded to the human brain.
  3. the resemblance that the computer bears to the human brain is purely mechanical
  4. the unintentional mixing up of word sequences in the computer can result in poetry.



Ans.d


Q.Computers have acquired a proven ability of performing many of the functions of the human brain because


  1. the brain of modern man is unable to discharge its functions properly on account of over-reliance on machines
  2. the sophisticated computer mechanism is on the verge of outstripping human mental faculties
  3. the process of organizing and communicating are similar in both cases
  4. the mechanics of the human brain have been introduced in the computer.



Ans.c


Q.The resemblance between the human brain and the computer is


  1. imaginary.
  2. intellectual.
  3. mechanical.
  4. functional.



Ans.c


Q.The passage implies that


  1. computers are assuming human forms
  2. human are assuming mechanical form
  3. computers and humans are substitutable
  4. there is continuity between what is human and what is mechanical.



Ans.d


Q.The author uses the word 'recognize' in relation to computer poetry to convey a


  1. sense of sorrow at the reluctant admission of the superiority of machines by mankind.
  2. feeling that computers have yet to conquer the emotional heights that man is capable of attaining
  3. feeling of derision for the popular faith in the omnipotence of the computer
  4. feeling of a fatalistic acceptance of the computer's encroachment upon human bastions



Ans.b


Q.Points of dissimilarity between the human brain and the computer don't extend to


  1. the faculty of composing poetry
  2. methods of communication
  3. the faculty of composing poetry
  4. the faculty of speaking naturally

Ans.b




Passage


A distinction should be made between work and occupation. Work implies necessity; it is something that must be done as contributing to the means of life in general and to one's own subsistence in particular. Occupation absorbs time and energy so long as we choose to give them; it demands constant initiative, and it is its own reward. For the average person the element of necessity in work is valuable, for he is saved the mental stress involved in devising outlets for his energy. Work has for him obvious utility, and it bring the satisfaction of tangible rewards. Where as occupation is an end in itself, and we therefore demand that it shall be agreeable, work is usually the means to other ends – ends which present themselves to the mind as sufficiently important to compensate for any disagreeableness in the means. There are forms of work, of course, which since external compulsion is reduced to a minimum, are hardly to be differentiated from occupation.



The artist, the imaginative writer, the scientist, the social worker, for instance, find their pleasure in the constant spontaneous exercise o creative energy and the essential reward of their work is in the doing of it. In all work performed by a suitable agent there must be a pleasurable element, and the greater the amount of pleasure that can be associated with work, the better. But for most people the pleasure of occupation needs the addition of the necessity provided in work. It is better for them to follow a path of employment marked out for them than to have to find their own.

When, therefore, we look ahead to the situation likely to be produced by the continued rapid extension of machine production, we should think not so much about providing occupation for leisure as about limiting the amount of leisure to that which can be profitably used. We shall have to put the emphasis on the work – providing rather than the goods – providing aspect of the economic process. In the earlier and more ruthless days of capitalism the duty of the economic system to provide work was overlooked. The purpose of competitive enterprise was to realize a profit. When profit ceased or was curtailed, production also ceased or was curtailed.

Thus the workers, who were regarded as units of labour forming part of the costs of production, were taken on when required and dismissed when not required. They hardly thought of demanding work as a right. And so long as British manufacturers had their eyes mainly on the markets awaiting them abroad, they could conveniently neglect the fact that since workers are also consumers, unemployment at home means loss of trade. Moral considerations did not yet find a substitute in ordinary business prudence. The labour movements arose largely as a revolt against the conception of workers as commodities to be bought and sold without regard to their needs as human beings. In a socialist system it is assumed that they will be treated with genuine consideration, for, the making of profit not being essential, central planning will not only adjust the factors of production to the best advantage but will secure regularity of employment.

But has the socialist thought about what he would do if owing to technological advance, the amount of human labour were catastrophically reduced? So far as I know, he has no plan beyond drastically lining the hours of work, and sharing out as much work as there may be. And, of course, he would grant monetary relief to those who were actually unemployed. But has he considered what would be the moral effect of life imagined as possible in the highly mechanized state of future? Has he thought of the possibility of bands of unemployed and under-employed workers marching on the capital to demand not income (which they will have) but work?




Q.Future, according to the passage, may find the workers


  1. without money
  2. without work
  3. replacing machines.
  4. without leisure

Ans.b


Q.The main defect of socialism at present is that


  1. it has not evolved a satisfactory system of making workers co-sharers in prosperity.
  2. it has not made work less burdensome for the mass of workers
  3. it has not taken into consideration the possibility of an immense reduction of human labour in the wake of mechanization.
  4. it is not concerned with improving and streamlining the method of production.

Ans.c




Q.The labour movement was the outcome of


  1. an effort to increase productivity
  2. a move to make workers share in the prosperity of the capitalists.
  3. a revolt against the conception of workers as commodities.
  4. a move to avert mass unemployment because of the mechanization

Ans.c




Q.The chief purpose of competitive enterprise is to


  1. create more job opportunities.
  2. produce as much as possible.
  3. create more wealth in the country.
  4. realize the maximum profit.

Ans.d


Q.In the situation created by the rapid extension of machine production, our object should be to


  1. make work as light as possible
  2. provide increased opportunities for interesting occupation.
  3. limit the amount of leisure to that which can be profitably used.
  4. produce more and more goods



Ans.c


Q.The activities of the artist, the writer, the scientist etc. may be considered to be occupations because


  1. they often does not have any utilitarian value
  2. external compulsion is reduced to a minimum and they are agreeable and require quite a lot of initiative
  3. they occupies time and energy only so long as the workers choose to give them
  4. they care only for the pleasure which brings them without any consideration of reward

Ans.b




Q.Which of the following statements is not true according to the information contained in the passage?


  1. Work is something done as contributing to the means of life in general and to one's own subsistence in particular.
  2. Occupation is something that requires initiative and can be done at one's will and pleasure and not as a task.
  3. Work brings in tangible rewards while occupation is not utilitarian
  4. There is no form of work which shows approximation to occupation

Ans.d




Q.The chief reason for a person taking up an occupation may be stated to be :


  1. a desire to make profit.
  2. an irresistible urge to do something uncommon
  3. a wish to do something useful to society.
  4. a desire to do something which requires initiative and doing it at his will and pleasure

Ans.d




Q.The distinction between work and occupation is as follows


  1. Work at all times is unpleasant and occupation is always agreeable
  2. In work there is an element of necessity which is totally wanting in occupation
  3. Work has obvious utility and brings tangible rewards, while occupation is an end in itself
  4. Work and occupation often seem to be so very much alike that no distinction can be made between them

Ans.c




Passage


If the more articulate members of a community formed a coherent and united class with a common interest, democracy would probably replace in to the rule of that intelligent, educated minority; even as it is, the democracies of the modern world are much closer to this fate than they are to the much-canvassed dangers of mob rule. Far from oppressing the cultured minority, or any other minorities, democracy gives more of them more scope to have their way than any other system does. This is the lesson of experience. It might also have been derived from an analysis of the concept of democracy, if the concept had been accurately analyzed.


Q.The word articulate here refers to


  1. the elite.
  2. people who are endowed with a native intelligence
  3. that class which is well educated.
  4. people who are endowed with clarity of speech.

Ans.c




Q.What emerges as the truth from a reading of the paragraph is that


  1. forms of government other than democracy give the mobs great scope for self expression
  2. democracy provides greater scope for mob rule.
  3. democracy provides greater scope for the rule of the minority.
  4. forms of government other than democracy give the educated minority greater scope for self expression

Ans.c


Q.Our appreciation of the virtues of the democratic system


  1. is the result of an illusory concept.
  2. is the result of our negative response to other forms of government
  3. is the result of a proven record of the success of democracy.
  4. is the result of centuries of accurate research on the theoretical aspects of democracy

Ans.c




Q.The wide scope that democracy offers to the minorities can be made known


  1. by our common sense
  2. by our political theories
  3. by our native intelligence.
  4. by proper analysis.

Ans.d


Q.The author seems to be


  1. a supporter of mob rule
  2. a supporter of democracy
  3. against intelligence in minorities
  4. analysing the flaws of democracy.

Ans.b


Q.The institution of democracy, in modern times


  1. is on the brink of extinction
  2. has become vulnerable to the dangers of proletariat rule
  3. should be prepared for the inevitability of mob rule
  4. has become prone to the rule of particular class of people

Ans.d




Passage


We have planned development with a view to raising standard of living of our teeming millions. Hence our economic development is inspired by social justice.


Q.Which of the following will weaken the argument?


  1. Without economic development standard of living cannot be raised
  2. Social justice implies economic prosperity
  3. Development cannot be planned
  4. None of these



Ans.c


Q.The argument is based on which of the following assumptions?
I. Social justice is our aim and economic development is the means.
II. There is overpopulation in India.
III. Economic development will lead to social justice.


  1. Only I
  2. Both I and II
  3. Both I and III
  4. Both II and III

Ans.c


Q.Which of the following will strengthen the argument?


  1. Social justice can be done by raising the standard of living
  2. Economic planning is necessary for every state.
  3. For economic development production should be increased
  4. None of these

Ans.a




Passage


We will have to take more interest in hydro-electric projects. As the prices of oil have increased, it has become vital that such renewable sources of energy are tapped.


Q.The assumption/assumptions of the argument is /are which of the following?
I. Hydro electric power is a renewable source of energy.
II. Hydro electric power is comparatively cheaper.


  1. Only I
  2. only II
  3. Both I and II
  4. Neither I nor II

Ans.c




Q.Which of the following will weaken the argument?


  1. Generation of hydroelectric power is more costly than oil.
  2. OPEC increased oil prices
  3. Without energy we cannot manage
  4. None of these.

Ans.a




Passage


There can be no civilization without music, dance or art, for one is not fully, vibrantly alive without them.


Q.The assumption/assumptions of the argument is /are which of the following?
I. Civilization and art are closely linked up.
II. If people are not full of life there can be no civilization.


  1. Only I
  2. Only II
  3. Both I and II
  4. Neither I nor II

Ans.b




Q.Which of the following would weaken the argument?


  1. Music is the life of man.
  2. Living persons like music
  3. Art has no relation with civilization
  4. None of these.

Ans.c


Q.Which of the following would strengthen the argument?


  1. Music, dance and art are human activities
  2. Only the vibrantly alive can contribute to civilization
  3. Music injects new life in man
  4. None of these.

Ans.b




Passage


It is sometimes mooted that there can be democracy in a two party system. That would be correct if politics were a game like cricket or football; but politics is not sports.


Q.Which of the following would strengthen the argument?


  1. Two party system functions well
  2. Politics is a dirty game
  3. Two political parties limit the choice of the voters
  4. None of these

Ans.c




Q.Which of the following would weaken the argument?


  1. The game of politics is played like any other game, for example, football.
  2. Politics is not a sport.
  3. Political parties struggle for power
  4. None of these.

Ans.a


Q.The assumption/assumptions of the argument is/are which of the following?
I. Politics is not a game.
II. Two party system is ideal for democracy.
III. Cricket is played by two teams.


  1. Only I
  2. Only II
  3. Only III
  4. I, II, III

Ans.c




Passage


A difficult readjustment in the scientist's conception of duty is imperatively necessary. As Lord Adrain said in his address to the British Association, “unless we are ready to give up some of our old loyalties, we may be forced into a fight which might end the human race”. This matter of loyalty is the crux. Hitherto, in the East and in the West alike, most scientists, like most other people, have felt that loyalty to their own state is paramount. They have no longer a right to feel this. Loyalty to the human race must take its place.



Everyone in the West will at once admit this as regards Soviet scientists. We are shocked that Kapitza who was Rutherford's favourite pupil, was willing when the Soviet government refused him permission to return to Cambridge, to place his scientific skill at the disposal of those who wished to spread communism by means of H-bombs. We do not so readily apprehend a similar failure of duty on our own side. I do not wish to be thought to suggest treachery, since that is only a transference of loyalty to another national state. I am suggesting a very different thing; that scientists the world over should join in enlightening mankind as to the perils of a great war and in devising methods for its prevention.

I urge with all the emphasis at my disposal that this is the duty of scientists in East and West alike. It is difficult duty, and one likely to entail penalties for those who perform it. Bu after all it is the labours of scientists which have caused the danger and on this account, if on no other, scientists must do everything in their power to save mankind from the madness which they have made possible. Science from the dawn of history, and probably longer, has been intimately associated with war. I imagine that when our ancestors descended from the trees they were victorious over the arboreal conservatives because flints were sharper than coconuts.

To come to more recent times, Archimedes was respected for his scientific defense of Syracuse against the Romans; Leonardo obtained employment under the Duke of Milan because of his skill in fortification, though he did mention in a postscript that he could also paint a bit. Galileo similarly derived an income from the Grant Duke of Tuscany because of his skill in calculating the trajectories of projectiles. In the French Revolution those scientists who were not guillotined devoted themselves to making new explosives. There is therefore no departure from tradition in the present day scientist's manufacture of A-bombs and H-bomb.



All that is new is the extent of their destructive skill.I do not think that men of science can cease to regard the disinterested pursuit of knowledge as their primary duty. It is true that new knowledge and new skills are sometimes harmful in their effects, but scientists cannot profitably take account of this fact since the effects are impossible to foresee.

We cannot blame Columbus because the discovery of the Western Hemisphere spread throughout the Eastern Hemisphere an appallingly devastating plague. Nor can we blame James Watt for the Dust Bowl although if there had been no steam engines and no railways the West would not have been so carelessly or so quickly cultivated. To see that knowledge is wisely used in primarily the duty of statesmen, not of science; but it is part of the duty of men of science to see that important knowledge is widely disseminated and is not falsified in the interests of this or that propaganda.Scientific knowledge has its dangers; but so has every great thing.

And over and beyond the dangers with which it threatens the present, it opens up, as nothing else can, the vision of a possible happy world, a world without poverty, without war, with little illness. And what is perhaps more than all, when science has mastered the forces which mould human character, it will be able to produce populations in which few suffer from destructive fierceness and in which the great majority regard other people, not as competitors, to be feared, but as helpers in a common task. Science has only recently begun to apply itself to human beings except in their purely physical aspect.



Such science as exists in psychology and anthropology has hardly begun to affect political behaviour or private ethics. The minds of men remain attuned to a world that is fast disappearing. The changes in our physical environment require, if they are to bring well being, correlative changes in our beliefs and habits. If we cannot effect these changes, we shall suffer the fate of the dinosaurs, who could not live on dry land. I think it is the duty of science – I do not say of every individual man of science – to study the means by which we can adapt ourselves to the new world.

There are certain things that the world quite obviously needs; tentativeness, as opposed to dogmatism in our beliefs: an expectation of co-operation, rather than competition, in social relations, a lessening of envy and collective hatred. These are things which education could produce without much difficulty. They are not things adequately south in the education of the present day.It is progress in the human sciences that we must look to undo the evils which have resulted from a knowledge of the physical world hastily and superficially acquired by populations unconscious of the changes in themselves that the new knowledge has made imperative. The road to a happier world than any known in the past lies open before us if atavistic destructive passion can be kept in leash while the necessary adaptations are made.

Fears are inevitable in our time, but hopes are equally rational and far more likely to bear good fruit. We must learn to think rather less of the dangers to be avoided than of the good that will be within our grasp if we believe in it and let it dominate our thoughts. Science, whatever unpleasant consequences it may have by the way, is in its very nature a liberator, a liberator of bondage to physical nature and, in time to come a liberator from the weight of destructive passion. We are on the threshold of utter disaster or unprecedented glorious achievement. No previous age has been fraught with problems so momentous and it is to science that we must look for happy issue.




Q.The duty of science, according to the author is :


  1. to realize the vision of a happy new world
  2. to pursue knowledge for its own sake
  3. to see that only such discoveries as conducive to the progress of humanity should be made
  4. to study the means by which we can adapt ourselves to the new world

Ans.d


Q.Archimedes, Leonardo and Galileo have been mentioned to substantiate the statement that


  1. science has always been intimately associated with war
  2. from ancient times science has played a leading part in the life of man
  3. all learning has flourished only under the patronage of royalty and eminent personages
  4. in the past pursuit of knowledge was done for its own sake

Ans.a


Q.The ground on which the author suggests that all scientists should join in educating mankind regarding the perils of a great war is that




  1. scientists being among the most learned among people, should take the lead in this process of education
  2. it is the work of scientists which has led to this perilous situation and so they should do something to undo the mischief.
  3. science has always been associated with war and in the fitness of things, scientists should take the lead in trying to end it.
  4. all others like politicians and soldiers have vested interest in perpetuating war and by elimination, scientists alone may be trusted to work for its abolition

Ans.b


Q.In modern times, the crux of the matter as far as scientists are concerned is that


  1. their loyalty to the state should be declared in no uncertain terms.
  2. a readjustment in the scientist's conception of duty is imperatively necessary
  3. they should not object to stringent control by the state over their activities
  4. they should assert their independence and refuse to subject themselves to any kind of control.

Ans.b




Q.The instance of Kaptiza cited by the author goes to prove that


  1. every scientist has his price
  2. in Soviet Russia, communists do not tolerate independent scientists.
  3. scientists, whether in the East or West, have hitherto felt that loyalty to their own state is paramount.
  4. scientists in the West have a higher sense of responsibility than their counterparts in the East

Ans.c


Q.Which among the following statements is not true according to the information provided in the passage?


  1. If there is no readjustment in the scientist's conception of duty, the extinction of the human race by war is a distinct possibility
  2. Up till now, scientists all over the world have felt that loyalty to their own state is paramount
  3. It is the labours of scientists which have caused the danger of annihilation of mankind
  4. The tradition up to now has been that scientists have been respected for their pursuit of knowledge and not for their part in devising potent weapons of destruction



Ans.d


Q.The duty of the scientist, according to the passage, is


  1. to further the interests of his state with as much devotion as possible
  2. to pursue knowledge regardless of the consequences of their discoveries and inventions
  3. to see that important knowledge is widely disseminated and is not falsified in the interests of propaganda
  4. to refuse to serve national interests

Ans.c


Q.The evils which have resulted from knowledge of the physical world can only be overcome by


  1. a more intensive pursuit of scientific knowledge
  2. making scientists more responsible to society
  3. adequate progress in the human sciences
  4. enlightening the general public about the evils

Ans.c




Q.Science may be considered a liberator in the sense that :


  1. ultimately it may bring the nations of the world together
  2. it may make man's life a great deal happier than what it is now
  3. it may free man from bondage to physical nature and the weight of destructive passions
  4. it may end the tyranny of age old beliefs and superstitions.

Ans.c




Passage


Humans have probably always been surrounded by their kin – those to whom they have been related by blood or marriage. But the size, the composition, and the functions of their families and kinship groups have varied tremendously. People have lived not only in the “nuclear family”, made up of just the parents and their offspring, which is standard in the West and has been found almost everywhere, they have lived in extended families and in formal clans; they have been “avunculocal”; they have been “ultrolateral”, they have been conscious of themselves as heirs of lineages hundred of generations deep. However constructed, the traditional kinship group has usually provided those who live in it with security, identity, and indeed with their entire scheme of activities and beliefs.

The nameless billions of hunter-gatherers who have lived and died over the past several million years have been embedded in kinship groups, and when people started to farm about ten thousand years ago, their universe remained centered on kinship. Now that there was a durable form of wealth which could be hoarded-grain–some families became more powerful than other; society became stratified, and genealogy became an important means of justifying and perpetuating status. During the past few centuries, however, in part of the world-in Europe and the countries that have been developing along European lines-a process of fragmentation has been going on. The ties and the demands of kinship have been weakening, the family has been getting smaller and, some say, less influential, as the individual, with a new sense of autonomy and with new obligations to himself (or, especially in the last decade and a half, to herself),has come to the foreground.

A radically different mental order-self-centered and traceable not to any single historical development as much as to the entire flow of Western history since at least the Renaissance has taken over. The political and economic effects of this rise in individual self-consciousness have been largely positive: civil rights are better protected and opportunities are greater in the richer, more dynamic countries of the West; but the psychological effects have been mixed , at best. Something has been lost: a warmth, a sanity, and a supportiveness that are apparent among people whose family networks are still intact. Such qualities can be found in most of the Third World and in rural pockets of the U.S., but in the main stream of post-industrial society the individual is increasingly left to himself, to find meaning, stability, and contentment however he can.

An indication of how far the disintegration of traditional kinship has advanced is that a surprising number of Americans are unable to name all four of their grandparents. Such people have usually grown up in stepfamilies, which are dramatically on the rise. So is the single – parent family-the mother-child unit, which some anthropologists contend is the real nucleus of kinship, having already contracted to the relatively impoverished nuclear family, partly as an adaptation to industrialization kinship seems to be breaking down even further. With the divorce rate in America at about fifty percent and the remarriage rate at about seventy five, the traditional Judeo-Christian scheme of marriage to one person for life seems to be shading into a pattern of serial monogamy, into a sort of staggered polygamy, which some anthropologists, who believe that we aren't naturally monogamous to begin with, see as “a return of normality”.

Still other anthropologists explain what is happening somewhat differently; we are adopting delayed system of marriage, they say, with the length of the marriage chopped off at both ends. But many adults aren't getting married at all; they are putting “self-fulfillment” before marriage and children and are having nothing further to do with kinship after leaving their parents' home; their family has become their work associate or their circle of best friends. This is the most distressing trend of all; the decline in the capacity of long-term intimate bonding.



Q.The traditional kinship group provides:


  1. Security
  2. Identity
  3. Entire scheme of activity
  4. All of the above



Ans.d



Q.Which of the following is indicative of the extent of disintegration of kinship groups?


  1. A large number of Americans are unable to name all four of their grandparents.
  2. Growing number of single-parent families
  3. Increase in the average age at which males get married.
  4. Both (a) and (b)

Ans.d



Q.Which of the following statements is not true?


  1. When people started to farm ten thousand years ago, kinship became less important
  2. Some families became more powerful than others after farming was initiated
  3. Genealogy became an important means of perpetuating status after the advent of farming.
  4. Stratification of society was a result of hunter – gatherers taking up farming.



Ans.a



Q.According to the author, what has been sacrificed with the rise in individual self-consciousness?


  1. Sanity
  2. Supportiveness
  3. Warmth
  4. All of the above

Ans.d



Q.The theme of the passage is which of the following?


  1. The impact of the deterioration of kinship of groups on third world countries
  2. The correlation between the decline of traditional kinship groups and stratification of society
  3. The changes that have occurred to kinship group pattern and the effect of those changes on the individuals
  4. The political and economic repercussions of the decline of the nuclear family.



Ans.c



Q.What does the author mean by serial monogamy?


  1. Judeo-Christian scheme of marriage.
  2. Marriage to one person for life
  3. A sequence of marriages and divorces.
  4. Delayed marriage.

Ans.c



Q.Which of the following statements cannot be inferred from the above passage?


  1. Smaller families are more autonomous and influential
  2. The rise of the individuals can largely be viewed as a western phenomenon
  3. A different mental order is in evidence and can be traced to the renaissance period.
  4. Mainstream post-industrial society would benefit from a resurgence of kinship groups.

Ans.a



Q.The word “genealogy” refers to:


  1. family history
  2. kinship groups
  3. family authority
  4. nuclear family



Ans.a



Q.According to the passage, the most distressing trend is:


  1. Many adults are putting “self fulfillment” before marriage and children and aren't getting married at all.
  2. The American divorce rate of 50 percent and remarriage rate of 75 percent
  3. The contraction of the nuclear family to the mother – child unit.
  4. The inability to develop lasting personal relationship

Ans.d



Q.According to the passage, which statement is not true of kinship group fragmentation?


  1. It is apparent that in Europe and countries developing along European lines a process of fragmentation has been taking place during the past few centuries.
  2. A self-centered mental order has replaced the earlier kin-centered mental order and it can be traced to a specific historical development.
  3. The political and economic benefits of the rise of the individuals have not been largely positive.
  4. Psychological effects of the rise of the individuals have been both positive and negative.

Ans.c


Passage


The core of modern doctoring is diagnosis, treatment and prognosis. Most medical schools emphasize little else. Western doctors have been analyzing the wheezes and pains of their patients since the 17 century to identify the underlying disease of the cause of complaints. They did it well and good diagnosis became the hall mark of a good physician. They were less strong on treatment. But when sulphonamides were discovered in 1935 to treat certain bacterial infections, doctors found themselves with powerful new tools. The area of modern medicine was born. Today there is a ever-burgeoning array of complex diagnostic tests, and of pharmaceutical and surgical methods of treatment. Yet what impact has all this had on health? Most observers ascribe recent improvements in health in rich countries to better living standards and changes in lifestyle. The World Health Organization cities the wide differences in health between Western and Eastern Europe.

The two areas have similar pattern of diseases: heart disease, senile dementia, arthritis and cancer are the most common cause of sickness and death. Between 1947 and 1964, both parts of Europe saw general health improve , with the arrival of cleaner water, better sanitation and domestic refrigerators. Since the mid 1960s, however, E. European countries, notable Poland and Hungary, have seen mortality rates rise and life expectancy fall. Why? The WHO ascribes the divergence to differences in lifestyle-diet, smoking habits, alcohol, a sedentary way of life (factors associated with chronic and degenerative diseases) rather than differences in access in modern medical care. In contrast, the huge sum now spent in the same of medical progress produce only marginal improvements in health. America devotes nearly 12% of its GNP to it high technology medicine, more than any other developed country. Yet, overall, Americans die younger, lose more babies and are at least as likely to suffer from chronic diseases.

Some medical producers demonstrably do work: mending broken bones, the removable of cataracts, drugs for ulcers, vaccination, aspirin for headaches, antibiotics for bacterial infections, techniques that save new born babies, some organ transplant, yet the evidence is scant for many other common treatments. The coronary bypass, a common surgical technique, is usually to overcome the obstruction caused by a blood clot in arteries leading to the heart. Deprived of oxygen, tissues in the heart might otherwise die. Yet, according to a 1988 study conducted in Europe, coronary bypass surgery is beneficial only in the short term. A bypass patient who dies within five years has probably lasted longer than if he had simply taken drugs.

But among those who get to or past five years, the drug-takers live longer than those who have surgery. An American study completed in 1988 concluded that removing tissue from the prostate gland after the appearance of (non-cancerous) growth, but before the growths can do much damage, does not prolong life expectancy. Yet the operation was performed regularly and cost Medicare, the federally – subsidized system for the elderly, over $1 billion a year. Though they have to go through extensive clinical trials, it is not always clear that drugs provide health benefits. According to Dr. Louise Russell, a professor of economics at Rutgers University, in New Jersey, although anti – cholesterol drugs have been shown in clinical trails to reduce the incidence of deaths due to coronary heart disease, in ordinary life there is no evidence that extend the individual drug taker's life expectancy. Medical practice varies widely from one country to another. Each year in America about 60 of every 100,000 people have a coronary by-pass;

In Britain about six Anti-diabetic drugs are far more commonly used in some European countries than others. One woman in five, in Britain, has a hysterectomy (removal of the womb) at some time during her life; In America and Denmark, seven out of ten do so.Why? If coronary heart problems were far commoner in America than Britain, or diabetes in one part of Europe than another, such differences would be justified. But that is not so. Nor do American and Danish women become evidently healthier than British ones. It is the medical practice, not the pattern of illness or the outcome, that differs. Perhaps American patients expect their doctors to “do something” more urgently than British ones? Perhaps American doctors are readier to comply? Certainly the American medical fraternity grows richer as a result. No one else seems to have gained through such practices.

To add injury to insult, modern medical procedures may not be just of questionable worth but sometimes dangerous. Virtually all drugs have some adverse side-effects on some people. No surgical procedure is without risk. Treatments that prolong life can also promote sickness: the heart attack victim may be saved but survive disabled. Attempts have been made to sort out this tangle. The “outcomes movement” born in America during the past decade, aims to lessen the use of inappropriate drugs and pointless surgery by reaching some medical consensus–which drug to give? whether to operate or medicate?–through better assessment of the outcome of treatments. Ordinary clinical trials measure the safety and immediate efficacy of products or procedures.

The outcomes enthusiasts try to measure and evaluate far wider consequences. Do patients actually feel better? What is the impact on life expectancy and other health statistics? And instead of relying on results from just a few thousand patients, the effect of treating tens of thousands are studied retrospectively. As an example of what this can turn up, the adverse side-effects associated with Opren, an anti-arthritis drug, were not spotted until it was widely used. Yet Dr. Arnold Epstein, of the Harvard Medical School, argues that, worthy as it may be, the outcomes movement is likely to have only a modest impact on medical practice. Effectiveness can be difficult to measure: patients can vary widely in their responses. In some, a given drug may relieve pain, in others not: is highly subjective.

Many medical controversies will be hard to resolve because of data conflict. And what of the promised heart-disease or cancer cures? Scientists accept that they are unlikely to find an answer to cancer, heart disease or degenerative brain illness for a long while yet. These diseases appear to be highly complex, triggered when a number of bodily functions go awry. No one pill or surgical procedure is likely to be the panacea. The doctors probably would do better looking at the patient's diet and lifestyle before he becomes ill than giving him six pills for the six different bodily failure that are causing the illness once he has got it. Nonetheless modern medicine remains entrenched. It is easier to pop pills than change a lifetime'' habits. And there is always the hope of some new miracle cure -–or some individual miracle. Computer technology has helped produce cameras so sensitive that they can detect the egg in the womb, to be extracted for test tube fertilization.

Bio-materials have created an artificial heart that is expected to increase life expectancy among those fitted with one by an average of 54 months. Bio-technology has produced expensive new drugs for the treatment of cancer. Some have proved life-savers against some rare cancers; none has yet had a substantial impact on overall death rates due to cancer. These innovations have vastly increased the demand and expectations of health care and pushed medical bills even higher – not lower, as was once hoped. Inevitably, governments, employers and insurers who finance health care have rebelled over the past decade against its astronomic costs, and have introduced budgets and rationing to curb them.



Just as inevitably, this limits access to health care: rich people get it more easily than poor ones. Some proposed solutions would mean no essential change, just better management of the current system. But others, mostly from American academics, go further, aiming to reduce the emphasis on modern medicine and its advance. Their trust is two headed: (i) prevention is better – and might be cheaper – than cure; and (ii) if you want high-tech, high-cost medicine, you (or your insurers, but not the public) must pay for it, especially when its value is uncertain. Thus the finance of health-care systems, private or public, could be skewed to favour prevention rather than cure. Doctors would be reimbursed for preventive practices, whilst curative measures would be severely rationed. Today the skew is all the other way: Governments or insurers pay doctors to diagnose disease and prescribe treatment, but not to give advise on smoking or diet. Most of the main chronic diseases are man-made.

By reducing environmental pollution, screening for and treating biological risk indicators such as high blood pressure, providing vaccination and other such measures – above all, by changing people's own behavior – within decades the incidence of these diseases could be much reduced. Governments could help by imposing ferocious “Sin taxes” on unhealthy products such as cigarettes, alcohol, maybe even fatty foods, to discourage consumption. The trouble is that nobody knows precisely which changes – apart from stopping smoking – are really worth putting into effect, let alone how. It is clear that people whose blood pressure is brought down have a brighter future than if it stayed high; It is not clear that cholesterol screening and treatment are similarly valuable. Today's view of what constitutes a good diet may be judged wrong tomorrow.

Much must change before any of these “caring” rather than “cure” schemes will get beyond the academic drawing-board. Nobody has yet been able to assemble a coherent preventive programme. Those countries that treat medicine as a social cost have been wary of moves to restrict public use of advanced and / or costly medical procedures, while leaving the rich to buy what they like. They fear that this would simply leave ordinary people with third-class medicine. In any case, before fundamental change can come, society will have to recognize that modern medicine is an imprecise science that does not always work: and that questions of how much to spend on it, and how, should not be determined almost incidentally, by doctor's medical preferences.



Q.The discovery of sulphonamides


  1. helped the doctors to diagnose better.
  2. led to better treatment of some bacterial infections
  3. eventually led to pharmaceutical and surgical methods of treatment
  4. None of the above

Ans.b



Q.The current medical practice as carried out in America benefits mostly the


  1. doctors
  2. rich
  3. biotechnology companies
  4. None of the above

Ans.d





Q.In some European countries anti-diabetic drugs are far more commonly used than others because


  1. the drugs are fairly easy to take
  2. more people in those countries suffer from diabetes than in others.
  3. medical practice in different countries varies.
  4. the sedentary way of life which marks their lifestyle results in more people becoming diabetic.

Ans.c



Q.Which of the following statements is false?


  1. Coronary by-pass operation is entirely ineffective.
  2. Drug taking is sometimes better than undergoing coronary by-pass surgery.
  3. Removing tissue from prostrate gland after non-cancerous growths appear is a risky operation.
  4. The American medicare is billed about a billion dollars annually for prostrate operations.

Ans.a



Q.Which of the following measures if undertaken under 'Care rather than Cure' movement could prove to be controversial?


  1. Sin' taxes on harmful substances such as tobacco and alcohol.
  2. Screening for high blood pressure
  3. Providing vaccinations
  4. Cholesterol screening.

Ans.d





Q.The outcomes movement could make a significant impact on medical practice if only


  1. the efficacy of all drugs could be tested fast.
  2. the results from just a few thousands patients could be relied on.
  3. the patient had responded uniformly to drugs and medical procedure
  4. pain could be easily relieved

Ans.c



Q.Modern cure are known to be expensive because


  1. bio-material are expensive
  2. employing biotechnological process in making medicines is an expensive process
  3. there is a huge demand for them but the supply is limited.
  4. None of these.

Ans.b



Q.The main objection to 'care rather cure' approach is that it


  1. might leave the poor to fend for themselves.
  2. will lead to confusion as far as the choice of medical technique to be followed
  3. is not possible to put together coherent preventive programme
  4. will lead to the neglect to curative techniques.

Ans.a


Passage


Smith did not invent economics. Joseph Schumpter observed that “The Wealth of the Nations” did not contain “a single analytic idea, principle or method that was entirely new”. Smith's achievement was to combine an encyclopaedic variety of insight, information and anecdote, and to distill from it a revolutionary doctrine. The resulting masterpiece is the most influential book about economics ever published. Remarkably, much of it speaks directly to questions that are still of pressing concern. The pity is that Smith's great book, like most classics (of 900 pages), is more quoted than read. All sides in today's debates about economic policy have conspired to peddle a conveniently distorted version of its idea. If his spirit is still monitoring events, it will undoubtedly have celebrated the collapse of communism.

But it must also long to meet the politicians who have taken charge of a fine reputation and not so fine profile. And put them right on one or two points. Today Smith is widely seen as intellectual champion of self-interest. This is a misconception. Smith saw no moral virtue in selfishness ; on the contrary he saw its dangers. Still less was he a defender of capital over labour (he talked of the capitalist's “mean rapacity”), of the rising bourgeoisie over the common folk. His suspicion of self – interest and his regard for the people as a whole come through clearly in one of his best-known remarks: “People of the same trade often meet together, even have merriment and diversion, but the conversation ends in a conspiracy against the public, or in some contrivance to raise prices.” Far from praising self-interest as a virtue, Smith merely observed it to be a driving economic force.

In “The Wealth of Nations” he explained how this potentially destructive impulse is harnessed to the social good. What is to prevent greedy producers raising their prices until their customers can afford to pay no more? The answer is competition. If producers raise their prices too high, they create an opportunity for one or more among them to profit by charging less and thus selling more. In this way competition tames selfishness and regulates prices and quality. At the same time it regulates quantities. If buyers want more bread and less cheese, their demand enables bakers to charge more and obliges cheese-mongers to charge less. Profits in bread-making would rise and profits in cheese-making would fall; effort and capital would move from one task to the other.

Through Smith's eyes, it is possible to marvel afresh at this fabulously powerful mechanism and to relish, as he did, the paradox of private gain yielding social good. Only more so, for the transactions that deliver a modern manufactured good to its customer are infinitely more complicated than those described by Smith. In his day, remember, the factory was still a novel idea: manufacturing meant pins and coats. A modern car is made of raw materials that have been gathered from all over the world, combined into thousands of intermediate products, sub-assembled by scores of separate enterprises.

The consumer need know nothing of all this, any more than the worker who tapped the rubber for the tyres knows or cares what its final use will be. Every transaction is voluntary. Self-interest and competition silently process staggering quantities of information and direct the flow of good. Services, capital and labour – just as in Smith's much simpler world. Far-sighted as he was, he would surely have been impressed. Mind you, modern man has also discovered something else.

With great effort and ingenuity, and the systematic denial of personal liberty, governments can supplant self-interest and competition, and replace the invisible hand of market forces with collective endeavour and a visible input- output table. The result is a five-year waiting list for Trabants.Because Smith was convinced that the market would, literally, deliver the goods, he wanted it, by and large, left alone. He said that governments should confine themselves to three main tasks: defending the people from the “violence and invasion of other independent societies”, protecting every member of society from the “injustice or oppression of every other member of it”; and providing “certain public works and certain public institutions, which it can never be for the interest of any individual, or small number of individuals, to erect and maintain.”

Each of these jobs arises because the market in some ways fails. In the first two cases-collective defence and the administration of justice - the failure is the so-called free-rider problem. People disguise what they are willing to pay for a service that must be provided to everybody or not at all; they want to consume it and let others meet the cost. However the third job the provision of “certain public works and certain public institutions” goes much wider. Indeed, to modern minds, it threatens to be all encompassing. It recognizes not only the free-rider problem but also other species of market failure notably, the effects of private transactions on third parties, or “externalities”.

Smith has in mind roads, public education, and help for the destitute. As it turned out, millions of teachers, nurses, firemen, postmen, rubbish collectors, bus drivers and 57,000 varieties of civil servant have since marched through this opening. Smith's thinking already seems to permit a great deal of government intervention. Add some modern economics and the floodgates open. For instance, theorists have shown that if just one price in an economy is different from price under competition, efficiency may require other every price to be somewhat distorted as well. Less government intervention, it seems to follow, cannot be assumed to be better. Competition itself has changed out of recognition.

Modern economies, it is said, are driven not by countless small producers, but by handful of giant enterprises and monopolistic trade unions. And the rapid pace of industrial change has made the externality of pollution for more obvious than before. Smith, admittedly, is a bit thin on global warming. Above all, many have forgotten something than Smith saw clearly: that every advantage granted by government to one part of the economy puts the rest at a disadvantage. Accordingly, he talked not of “intervention” -a too-neutral word-but of “preference” and “restraint”. Modern governments offer preference as though it costs nothing: the beneficiaries demand it as of right. But Smith went further than revealing the penalty in every preference. He also understood that ministers,like markets, fail.

A great virtue of unfettered competition, he said, was that “the sovereign is completely discharged from a duty, in the attempting to perform which he must always be exposed to innumerable delusions, and for the proper performance of which no human wisdom or knowledge could ever be sufficient. “ Many of the reasons why markets fail are also reasons why governments fail at the same task. If the consumer refuses to reveal his preferences in a market setting, how are governments to discover them? All too often, moreover, government intervention is itself a cause of the market breaking down which becomes the reason for further rounds of intervention, and so on. In Britain think of tax preferences for housing, rent controls, planning, regulations;

America think of tax preferences for borrowing, deposit insurance, leverage buy-outs, financial-market regulation. In one crucial respect, Smith's arguments are even more powerful now than in this day. Naturally, he favoured free trade to prevent market failure: “By means of glasses, hotbeds, and hotwalls, very good grapes can be raised in Scotland, and very good wine too can be made of them at about thirty times the expense for which at least equally good can be brought from foreign countries. Would it be a reasonable law to prohibit the importation of all foreign wines, merely to encourage the making of claret and burgundy in Scotland?” Two centuries later, free trade is not just a matter of the cheapest supply; it is also the best way to force producers that might otherwise be near-monopolies to compete. It is perfect folly to complain that today's big companies render the invisible hand powerless, and to conclude that barriers to trade must go up: trade and competition need each other more than ever before. Smith was a pragmatist.



The principles he expounded on the proper role of government are flexible if anything, too flexible. They are a remainder that imperfect markets are usually cleverer than imperfect governments, but they cannot draw a line to separate good intervention from bad. If governments and voters could be guided by two Smithian precepts, however, the market system that has worked so well would work even better. First, the competitive clash of self interest against self-interest, however imperfect, has built-in safeguards. Before governments exert their monopoly power to displace it, they must justify themselves. Let the burden of proof always be on them, Second, when preference or restraint are judged to be necessary, use market forces to apply them. Tariffs are better than quotas; taxes are better than bans or direct controls; allocating resources by price (e.g. in health or education) is better than allocating them by fiat, even if the services are then provided “free” (but never forget those inverted commas) to their consumers.



Q.Smith's attitude to the virtues of self interest can be best described as


  1. pragmatic
  2. cynical
  3. skeptical
  4. supportive

Ans.a



Q.According to Adam Smith


  1. selfishness is dangerous.
  2. competition is the result of 'mean rapacity
  3. self interest always leads to competition
  4. competition regulates quantities

Ans.d



Q.All of the following are reasons for market failure except:


  1. The effects of private transactions on third parties.
  2. People would like to consume goods without paying for them.
  3. Unfettered and unbridled trade.
  4. Government intervention.

Ans.c



Q.Adam Smith is most likely to agree with the statement:


  1. It is necessary for capital to exploit labour if competition and low prices are to be engendered.
  2. Businessmen would form cartels given the chance
  3. Lesser government intervention is better
  4. Collective endeavour could be the basis of economic growth.

Ans.b





Q.Which of the following situations is not an instance of market failure?


  1. A government practising apartheid.
  2. A specialist doctor charging high fees.
  3. Poor development of roads and railways
  4. A murderer going scot-free.

Ans.d



Q.The 'free rider' problem results in the need for all of the following except


  1. government laws to prevent crime.
  2. a national defence budget.
  3. a national R & D centre for an industry.
  4. a United Nations peace keeping force.

Ans.c



Q.Based on the passage, competitions will directly affect all of the following except:


  1. quantity of a good produced.
  2. quality of a good produced
  3. direction of flow of goods.
  4. price of goods sold.

Ans.b



Q.We can conclude from the passage that


  1. government control is often self propagating
  2. rulers are prone to delusions
  3. governments often fail because markets also fail
  4. government actions rarely have justifications.

Ans.a





Q.Based on the passage, we could say that Adam Smith would not support


  1. government intervention
  2. corporation
  3. taxes
  4. import licences.

Ans.d



Q.All the following characteristics of the modern world are used as arguments for government intervention except.


  1. advanced and costly research in basic science.
  2. the far greater complexity of the modern manufacturing process.
  3. increased pollution and environmental hazards.
  4. the pre-eminence of large corporations.

Ans.b



Q.Based on the passage, the following can be inferred, except which of the following?


  1. Governments must act only when necessary.
  2. High customs duties are an acceptable way to restrict a change.
  3. High taxation is better than bans.
  4. The role of governments must be more flexible.

Ans.d



Q.The most serious problem of modern government is that they


  1. hire too many people.
  2. offer advantages to groups as if it costs nothing.
  3. are often unwise in their decisions.
  4. tax the citizens to much.

Ans.b


Passage


The communities of ants are sometimes very large, numbering even up to 500, individuals: and it is a lesson to us that no one has ever yet seen quarrel between any two ants belonging to the same community. On the other hand, it must be admitted that they are in hostility not only with most other insects, including ants of different species, but even with those of the same species if belonging to different communities. I have over and over again introduced ants from one of my nests into another nest of the same species; and they were invariably attacked, seized by a leg or an antenna, and dragged out.

It is evident, therefore, that the ants of each community all recognize one another, which is very remarkable. But more than this, I several times divided a nest into two halves and found that even after separation of a year and nine months they recognize one another and were perfectly friendly, while they at once attacked ants from a different nest, although of the same species. It has been suggested that the ant of each nest have some sign or password by which they recognize one another. To test this I made some of them insensible, first I tried chloroform; but this was fatal to them, and I did not consider the test satisfactory. I decided therefore to intoxicate them. This was less easy than I had expected.

None of my ants would voluntarily degrade themselves by getting drunk. However, I got over the difficulty by putting them into whisky for a few moments. I took fifty specimens - - twenty five percent from one nest and twenty five percent from another made them dead drunk, market each with a spot of paint, and put them on a table close to where other ants from one the nests were feeding. The table was surrounded as usual with a moat of water to prevent them from straying. The ants, which were feeding, soon noticed those, which I had made drunk.

They seemed quite astonished to find their comrades in such a disgraceful condition, and as much at a loss to know what to do with their drunkards as we were. After a while, however, they carried them all away; the strangers they took to the edge of the moat and dropped into the water, while they bore their friends home into the nest, where by degrees they slept off the effects of the spirits. Thus it is evident that they know their friends even when incapable of giving any sign or password


Q.An appropriate title for this passage might be


  1. Nature's Mysteries
  2. Human Qualities in the Insect world
  3. Drunken Ants
  4. Communication in Ant Communities

Ans.d


Q.Attitudes of ants towards strangers of the same species may be categorized as


  1. indifferent
  2. curious
  3. hostile
  4. passive

Ans.c


Q.The author's anecdotes of the inebriated ants would support all the following inductions except the statement that


  1. ants take unwillingly to intoxicants
  2. ants aid comrades in distress
  3. ants have invariable recognition of their community members
  4. ants recognize their comrades by a mysterious password

Ans.d


Q.According to the passage, chloroform was less successful than alcohol for inhibiting communication because of


  1. its expense
  2. its unpredictable side effects
  3. its unavailability
  4. its fatality

Ans.d


Q.Although the author is a scientist, his style of writing also exhibits a quality of


  1. sophistry
  2. whimsy
  3. hypocrisy
  4. tragedy

Ans.b


Passage


Compared with other experimental sciences, astronomy has certain limitations. First, apart from meteorites, the Moon, and the nearer planets, the objects of study are inaccessible and cannot be manipulated, although nature sometimes provides special conditions, such as eclipses and other temporary effects. The astronomer must content himself with studying radiation emitted or reflected from celestial bodies. Second, from the Earth's surface these are viewed through a thick atmosphere that completely absorbs most radiation except within certain “windows”, wavelength regions in which the radiation can pass through the atmosphere relatively freely in the optical, near-infrared, and radio bands of the electromagnetic spectrum;

and even in these windows the atmosphere has considerable effects. For light, these atmospheric effects are as follows: (1) some absorption that dims the radiation somewhat, even in a clear sky; (2) refraction, which causes slight shift in the direction so that the object appears in a slightly different place; (3) scintillation (twinkling); i.e., fluctuations in brightness of effectively point – like sources such as stars, fluctuations that are, however, averaged out for objects with larger images, such as planets (the ionosphere, an ionized layer high in the atmosphere, and interplanetary medium have similar effects on radio sources);

(4) image movement because of atmospheric turbulence (“bad seeing”) spreads the image of a tiny point over an angle of nearly one arc second or more on the celestial sphere (one arc second equals 1/3, 600 degrees); and (5) background light from the night sky. The obscuring effects of the atmosphere and its clouds are reduced by placing observing stations on mountains, preferably in desert regions (e.g., southern California and Chile), and away from city lights.

The effects are eliminated by observing from high-altitude aircraft, balloons, rockets, space probes, and artificial satellites. From stations all or most of the atmosphere, gamma rays and X-rays-that is, high-energy radiation at extremely short wave-lengths and far-ultraviolet rays and far-infrared radiation, all completely absorbed by the atmosphere at ground level observatories can be measured, At radio wave-lengths between about one centimeter and 20 meters, the atmosphere (even when cloudy) has little effect, and man-made radio signals are the chief interference.

Third, the Earth is a spinning, shifting, and wobbling platform. Spin on its axis causes alternation of day and night and an apparent rotation of the celestial sphere with stars moving from east to west. Ground – based telescopes use a mounting that makes it possible to neutralize the rotation of Earth relative to the stars; with an equatorial mounting driven at a proper speed, the direction of the telescope tube can be kept constant for hours while the Earth turns under the mounting. Large radio telescopes usually have vertical and horizontal axes (altazimuth mounting), with their pointing continuously controlled by a computer.

In addition to the daily spin, there are much more gradual effects, called precession and nutation. Gravitational action of the Sun and Moon on the Earth's equatorial bulge causes the Earth's axis to process like a top or gyroscope, gradually tracing out a circle on the celestial sphere in about 26,000 years, and also to nutate or wobble slightly in a period of 18.6 years. The Earth's rotation and orbital motion provide the basic standard of directions of stars, so that uncertainties in the rate of these motions can lead to quite small but important uncertainties in measurements of stellar movements.


Q.One of the type of radiations that cannot pass through the atmospheric 'windows' without distortion is


  1. near infra-red spectrum.
  2. far-ultraviolet spectrum.
  3. optical band in the spectrum.
  4. radio band in the spectrum

Ans.b


Q.One of the atmospheric effects earth – based experiments that is not mentioned in the passage is


  1. twinkling
  2. refraction.
  3. image movement.
  4. clouds from volcano eruptions.

Ans.d


Q.The purpose of telescope mounting is to neutralize


  1. atmospheric interference
  2. the effect of precession.
  3. the effect of nutation.
  4. the effect of diurnal spinning

Ans.d


Q.The precession period of Earth is


  1. 24 hours
  2. 365.25 day
  3. 18.6 years
  4. 26,000 years

Ans.d


Q.Gravitational action of the Sun and the Moon on Earth causes
I. diurnal spinning
II. Precession
III. Nutation


  1. I only
  2. I and II only
  3. II and III only
  4. I, II and III

Ans.c


Q.The orbital motion of the Earth


  1. is partly caused by the moon
  2. can have uncertain rates.
  3. has a periodicity of 18.6 years
  4. is neutralized by telescope mounting.

Ans.b


Q.The man-made radio signals have wave-lengths of


  1. more than 20 meters
  2. less than one centimeter.
  3. between one centimeter and 20 meters
  4. gamma rays

Ans.c


Passage


If American policy towards Europe in the postwar years had been a conspicuous success, and towards Asia a disappointing balance between success and failure, it could be said that the most conspicuous thing about relations with Latin America was the absence of any policy. Franklin Roosevelt, to be sure, had launched a “Good Neighbour” policy, but being a good neighbour was, it seemed, a negative rather than a positive affair, a matter of keeping hands off, of making the Monroe Doctrine, in form at least, multilateral.

All through the postwar years, the states of Latin America - - Mexico and Chile were partial exceptions - were in the throes of major economic and social crises. Population was growing faster than in any other part of the globe, without a comparable increase in wealth or productivity; the gap between the poor and the rich was widening; and as the rich and powerful turned to the military for the preservation of order and privilege, the poor turned to revolution. Deeply involved in other quarters of the globe, the United States paid little attention to the fortunes or misfortunes of her neighbours to the south, and when she did intervene, it appeared to be on the side of order and the status quo rather than on the side of reform.

So frightened was the United States of “Communism” in Latin America that it preferred military dictatorship to reformers who might drift too far to the “left”, and sustained a Batista in Cuba, a Trujillo in the Dominican Republic, a Peron in Argentina, and a Jimenez in Venezuela. In his last two years, President Eisenhower had tried to mend his Latin American fences. Though rejecting a Brazilian proposal of a Marshall Plan for Latin America, he did take the initiative in setting up an InterAmerican development Bank with a capital of one billion dollars, almost half of it supplied by the United States. Other government investments in Latin America ran to some four million dollars, while private investments exceeded nine billion.

Yet though to most Americans, all this seemed a form of economic aid, many Latin Americans regarded it as economic imperialism. In September 1960, came a co-operative plan that could not be regarded as other than enlightened: the Act of Bogota, which authorized a grant of half a billion dollars to subsidize not only economic but social and educational progress in Latin America. “We are not saints”, said President Eisenhower when he visited Santiago de Chile, “We know we make mistakes, but our heart is in the right place”.

But was it? President Kennedy was confronted by the same dilemma that had perplexed his predecessors. Clearly it was essential to provide a large-scale aid to the countries south of Rio Grande, but should this aid go to bolster up established regimes and thus help maintain status quo, or should it be used to speed up social reforms, even at the risk of revolt? As early as 1958, the then Senator Kennedy had asserted that “the objective of our aid program in Latin America should not be to purchase allies, but to consolidate a free and democratic Western Hemisphere, alleviating those conditions which might foster opportunities for communistic infiltration and uniting our peoples on the basis of constantly increasing living standards”.

This conviction that raising the standards of living was the best method of checking Communism now inspired President Kennedy's bold proposal for the creation of the alliance for progress - - a ten year plan designed to do for Latin America what Marshall Plan had done for Western Europe. It was to be “a peaceful revolution on a hemispheric scale, a vast co-operative effort, unparalleled in magnitude and nobility of purpose, to satisfy the basic needs of the American people for homes, work, land, health and schools. “To achieve this, the United States pleaded an initial grant of one billion dollars, with the promise of additional billions for the future.


Q.Following World War II, which problem was the United States most concerned with regarding Latin America?


  1. Economic stability.
  2. Political ideology.
  3. Religious persecution.
  4. Military dictatorship.

Ans.b


Q.A key reason why Latin Americans rejected the Inter-American development Bank was that


  1. it primarily provided money for social reform subsidies.
  2. the moneys provided were only for specific performance projects
  3. it constituted an extension of the Marshall Plan into Latin America
  4. it was being used as a means to control the economic destiny of Latin America

Ans.d


Q.Which of the following is most closely associated with the concept of a Marshall Plan for Latin America?


  1. The Good Neighbour Policy.
  2. The Alliance for Progress.
  3. The Act of Bogota.
  4. The Monroe Doctrine.

Ans.a


Q.According to the passage, the fundamental change in U.S. foreign policy directed towards Latin America


  1. resulted in a deterioration of U.S. Latin American relations.
  2. was responsible for Peron remaining as a dictator in Peru.
  3. recognized that economic aid alone would prevent social revolutions.
  4. provided for increased military and economic aid to prevent the spread of communism in Latin America.

Ans.c


Q.Which of the following statements is not true?


  1. Mexico and Chile did not experience the general social crises that are common to the majority of Latin American countries.
  2. President Eisenhower continued in practice the theory that economic aid was the best defense against communist incursion into Latin America
  3. The Good Neighbour Policy favoured a multilateral interpretation of the Monroe Doctrine
  4. The traditional U.S. approach in Latin America was to protect the status quo.

Ans.b


Q.Which of the inferences can be drawn if everything said in the passage were assumed to be true?


  1. Rebellions are fuelled by social reforms and avoided by supporting established authorities or continuing the present state of affairs.
  2. The American policy towards Asia can be called an overall success, though small in magnitude.
  3. Kennedy, in 1958, wanted America to aid South American countries to acquire more support in their fight against communism.
  4. Eisenhower rejected the Marshall Plan, whereas Kennedy implemented a similar one.

Ans.a


Passage


In order to better understand conservation in China, it is essential that one has a grasp of what the term “Chinese conservatism” means. Chinese conservatism is markedly different from the conservatism of the modern West. The political term “conservative” came about during the French Revolution and inspired men who were determined to preserve Christian and aristocratic elements in European society.

Chinese conservatism began around the time of the Taiping Rebellion and had as its primary objectives the preservation of both Confucian society and non-feudal strains of pre-Opium War Chinese society. While western conservatism believes in sacredness of private property and distrust of cosmopolitanism, the Chinese conservatism is the defense of a rational cosmopolitan order. Thus, the only common area of agreement between European and Chinese conservatism is the intent to conserve. During the Tung-chin Restoration, the great aim was the revival of Confucian values and institutions.

But these aims had to be modified so that they might endure. Restoration statesmen had no desire to create a new society – they wanted to restore a society that they believed had been based on truth. The statesmen of the Restoration stretched the traditional ideology to its limits in an effort to make the Confucian system under new conditions. They were true conservatives in a great tradition, living in an age when revolutionary change was unavoidable. The aim of the Restoration was to restore to their original vitality the best of the ancient institutions.

During the Restoration, the two immediate problems were the suppression of rebellion and the stabilization of foreign relations. In addition, the people were striving for a restoration of the system of government by superior civil officials. The men in the hierarchy of the Restoration rose to prominence through proven ability in both civil and military affairs. They emphasized human and social training – that is, indoctrination, morality, and the art of leadership through the cultivation of character. The great majority of the officials rose through the examination system. During the chaos of this period, the examination system had lost much of its effectiveness. This is important and must be noted because the examination system was the traditional avenue for selecting officials.

The senior official of Restoration realized that their policies would be ineffective unless the quality of the junior official was improved, so it was their duty to weed out the officials who had attained office in irregular ways and to promote the examination system as the only way to high position. But these men of the Restoration had enough foresight to determine that it was impossible to select officials automatically on the basis of objective tests alone. As a result, the system of recommendation was ushered in, whereby; a high official sponsored the career of a promising young man. This acted as an important supplement to the examination system.


Q.The traditional method for selecting officials was


  1. approximately by the civil government.
  2. the examination system
  3. through a subjective testing system.
  4. sponsorship by a high government official.

Ans.b


Q.A primary objective in the development of Restoration thought was


  1. to modify traditional Chinese society to reflect new conditions
  2. to create a new society based on truth.
  3. the knowledge that Chinese conservatism is superior to western conservatism.
  4. the desire to familiarized China with military technology.

Ans.a


Q.The major similarity between Chinese and western conservatism is


  1. that Chinese conservatism attempted to preserve traditions
  2. that Chinese conservatism developed during the Taiping Revolution.
  3. the cosmopolitan nature of western conservatism
  4. that Chinese conservatism is primarily land oriented

Ans.a


Q.The most significant Chinese philosopher mentioned in the passage is


  1. Tung-chin.
  2. I. Ching.
  3. Buddha
  4. None of the above.

Ans.d


Q.During the Restoration, ancient institutions


  1. were no longer accepted as a viable alternative to western technology.
  2. were studied only as classical examples of a former glorious past.
  3. were to be the cornerstones of a changing but traditional society
  4. were considered as a primary reason for the decline of traditional China

Ans.c


Q.The western conservatives intended to preserve all the following except


  1. Christianity.
  2. private property.
  3. cosmopolitanism.
  4. aristocratic elements

Ans.c


Q.The most appropriate title for the passage will be


  1. The Chinese examination system
  2. Chinese Conservatism
  3. How the officials rose
  4. Impact of the Taiping Rebellion

Ans.b


Passage


Every state has a constitution, since every state functions on the basis of certain rules and principles. It has often been asserted that the United States has a written constitution, but that the constitution of Great Britain is unwritten. This is true only in the sense that, in the United States, there is a formal document called the Constitution, whereas there is no such document in Great Britain. In fact, however, many parts of the British constitution exist in written form, whereas important aspects of the American constitution are wholly unwritten.

The British constitution includes the bill of Rights (1689), the Act of Settlement (1700 – 01), the Parliament Act of 1911, the successive Representation of the People Acts (which extended the suffrage), the statutes dealing with the structure of the courts, the various local government acts, and many others. These are not ordinary statutes, even though they are adopted in the ordinary legislative way, and they are not codified within the structure of single orderly document. On the other hand, such institutions in the United States as the presidential cabinet and the system of political parties, though not even mentioned in the written constitution, are most certainly of constitutional significance.

The presence or absence of a formal written document makes a difference, of course, but only one of degree. A singledocument constitution has such advantages as greater precision, simplicity, and consistency. In a newly developing state as Israel, on the other hand, the balance of advantage has been found to lie with an uncodified constitution evolving through the growth of custom and the medium of statutes. Experience suggests that some codified constitutions are much too detailed.

An overlong constitution invites disputes and litigation is rarely read or understood by the ordinary citizen and injects too much rigidity in cases in which flexibility is often preferable. Since a very long constitution says to many things on too many subjects, it must be amended often, and this makes it still longer. The United States Constitution of 7,000 words is a model of brevity, whereas many of that country's state constitutions are much too long - the longest being hat of the sate of Louisiana, whose constitution now has about 255,000 words.

The very new, modern constitutions of the recently admitted states of Alaska and Hawaii and the Commonwealth of Puerto Rico have, significantly, very concise constitutions ranging from 9,000 to 15,000 words. The 1949 constitution of India, with 395 articles, is the wordiest of all national constitutions. In contract, some of the world's new constitutions, such as those of Japan and Indonesia, are very short indeed.

Some constitutions are buttressed by powerful institutions such as an independent judiciary, whereas other, though committed to lofty principles, are not supported by governmental institutions endowed with the authority to defend these principles in concrete situation. Accordingly, many juristic writers distinguish between “normative” and “normal” constitutions. A normative constitution is the one that not only has the status of supreme law but it also fully activated and effective; it is habitually obeyed in the actual life of the state. A nominal constitution may express high aspirations, but it does not, in fact, reflect the political realities of the state.

Article 125 of the 1936 constitution of the Soviet Union and the article 87 of the 1954 constitution of the People's Republic of China both purport to guarantee freedom of speech, but in those countries even mild expressions of dissent are likely to be swiftly and sternly repressed. Where the written constitution is only nominal, behind the verbal façade will be found the real constitution containing the basic principles according to which power is exercised in actual fact. Thus in the Soviet Union, the rules of the Communist Party describing its organs and functioning are more truly the constitution of that country than are the grand phases of the 1936 Stalin constitution. Every state, in short has a constitution, but in some, real constitution operates behind the façade of a nominal constitution.


Q.The lengthiest constitution in the world is that of


  1. Great Britain
  2. India
  3. Puerto Rico
  4. Soviet Union.

Ans.b


Q.The instance of a country without a written constitution mentioned in the passage is


  1. People's Republic of China
  2. Japan
  3. Israel
  4. Indonesia

Ans.c


Q.The unwritten parts of the US constitution deal with


  1. Courts
  2. presidential cabinet.
  3. relationship between the Centre and the States
  4. fundamental rights.

Ans.b


Q.In the United States


  1. the newly admitted states have lengthy constitutions.
  2. the newly admitted states have concise constitutions
  3. the political parties have no constitutional significance.
  4. the constitution can be termed 'normal'

Ans.b


Q.In countries with 'normative' constitutions


  1. there will be very little freedom of speech
  2. there are effective instruments to enforce their provisions.
  3. political realities are different from what are enshrined in them.
  4. there are frequent amendments to them

Ans.b


Q.By 'normal' constitution, the author means


  1. written constitution
  2. one that contains lofty ideals
  3. a lengthy constitution.
  4. a constitution that is not being enforced.

Ans.d


Q.One of the drawbacks of a long constitution is


  1. its publication is expensive.
  2. it is difficult to understand.
  3. it may require to be amended frequently
  4. it is difficult to enforce

Ans.c


Q.According to the author, the difference between a written and an unwritten constitution


  1. has no significance.
  2. is just one of degree
  3. has been exaggerated by politicians
  4. cannot be defined.

Ans.b


Passage


An urgent problem is now threatening libraries throughout the world. Their collections, which are crucial for diverse purposes as economic development, educational research and recreational pursuits, are in danger of disintegrating. The problem is mainly due to one cause – the type of paper on which books have been printed for the past one and a half centuries. Until the 1850s, paper was produced from linen or cotton rags and proved to be relatively long-lasting. In the mid-19th century, however, the popular demand for paper and the commercial need for an economic method of production led to the use of mechanically ground wood pulp. Paper manufactured for wood pulp is highly acidic and therefore inherently unstable. It contains lignin – a major factor in causing paper to discolour and disintegrate. The useful lifespan of most 20th-century book papers has been estimated to be no more than a few decades. Libraries comprise an important part of the market for printed books and they are increasingly aware of the fragility of this material. The extent of the deterioration of library collections is alarming. Surveys conducted at various major institutions reveal that 26% to 40% of the books they hold are seriously embrittled and thus unavailable for normal use. Programmes are now being developed with two main aims in mind – on the one hand, to improve the physical condition of library collections, especially by the process called 'mass de-acidification' (which is designed to eliminate acid from the paper of published books and insert a buffer compound that will provide protection against future acid attack from the environment); and on the other, to transfer the contents of existing books to another medium (such as microfilm or optical disk). Libraries will only be able to carry out these special tasks with the assistance of other experts such as book conservators and high-technology specialists. But here is another group with whom librarians have traditionally enjoyed strong affinities and whose co-operation will be crucial if the problem of decaying collections is to be arrested – namely, the printing and publishing industries. The existing problem – that of book collections already assembled in libraries – is of vast proportions, but it is intensified by the continuing use of acid-based paper in book publishing. The key issue is how to preserve the books of the future, not simply those of the past. If the future dimensions of the conservation problem are to be curbed, there will need to be widespread adoption of paper which is of archival quality. This change does not relate to a narrowly perceived need because the long term preservation of library collections is important – both for the overall social benefits they bring as well as for the special advantages they bestow on the printing and publishing industries. In the first place, libraries are of critical importance to the future well-being of citizens since they provide the knowledge base of society. They contain the record of humanity – the accumulation of ideas and insights and discoveries on which social effort and progress are possible. The destruction of libraries would represent an immense cultural loss, a form of amnesia which would affect every member of society. In the second place, printers and publishers have an economic interest in turning to paper of archival quality. So long as the libraries are acquiring books with a short lifespan they will be forced to devote an increasing share of their budgets to conservation. These budgets are severely strained by the combined impact of inflation and currency devaluation, and there is scarcely any prospect of enlarged government funding. As a result, libraries will be compelled to balance the preservation of their collections against the expansion of those collations. In short, the choice will be between conservation and acquisition – and the funds for conservation are likely to come from acquisition budgets. This unpalatable choice will damage both libraries and the printing and publishing industries and can only be minimized in its effects by a bold decision to convert to use of permanent paper.


Q.The tone of the passage is one of


  1. informed concern
  2. destructive criticism.
  3. derisive ridicule
  4. helpless alarm.

Ans.a


Q.The phrase 'archival quality' implies


  1. a smooth paper.
  2. thick paper.
  3. long-lasting paper.
  4. alkaline paper.

Ans.c


Q.Wood-pulp as raw material for paper was developed because of


  1. the need to produce large quantities of paper.
  2. the shortage of linen
  3. the need to develop non-acidic paper.
  4. scientific research.

Ans.a


Q.If paper has to last long …


  1. it should be made of cotton rags.
  2. it should be non-acidic.
  3. it should be alkaline.
  4. preservatives must be used

Ans.b


Q.On of the reasons not mentioned in the passage in favour of producing long-lasting paper is


  1. it will help preserve the knowledge-base of society.
  2. it will enable more books to be brought by libraries.
  3. it will lead to more governmental allocation to libraries.
  4. it will help the publishing industry.

Ans.c


Q.Purchase of new books by libraries are bound to be curtailed because of all the following reasons except


  1. drastic reduction in governmental funding
  2. the need for spending more money for conservation of old books.
  3. the need to microfilm books.
  4. inflationary trends.

Ans.a


Q.Continued use of wood-pulp paper in book will affect
I. libraries.
II. General public.
III. the publishing industry.
IV. The governments.


  1. I and III only
  2. II and III only
  3. I, II, III and IV
  4. I, II, and III only

Ans.d


Q.The substance which causes paper to discolour is


  1. acid
  2. linen.
  3. lignin.
  4. preservatives.

Ans.c


Passage


The Japanese want their Emperor to reign for long, very long, but their Prime Ministers to have very short tenures. During the 61 years Hirohito has been on the Chrysanthemum throne, 38 Prime Ministers have come and gone (or at least 32, if returns to power are left out of account). Eisaku Sato's eight uninterrupted years as Prime Minister in the Sixties and early Seventies provoked fears about the possible ill-effects of one-man leadership on Japanese democracy, and led the dominant Liberal Democratic Party (LDP) to lay down the norm of a two-year for a party chief and head of Government. Mr.Yasuhiro Nakasone, now bowing out, has served for an unusual five years. His success as Prime Minister was evidenced by the ruling party re-electing him leader more than once. But his plan to push through the Diet a Bill to levy a 5% indirect tax as part of financial reforms failed, in spite of the LDP majority in both the chambers.

It was time then for him to go. The quick turnover of Primate Minister has contributed to the functioning of the LDP through factions. In the party that has ruled Japan for 32 years continuously, factionalism is not something unseemly. The leader is chosen by hard bargaining – some foreigners call it horse-trading– among the faction leaders, followed, if necessary, by a party election. For the decision in favour of Noboru Takeshita as the next President of the LDP and Primate Minister of Japan, voting was not necessary. His hopes were stronger than those of he other two candidates – Finance Minister Kiichi Miyazawa and former Foreign Minister, Shintaro Abe – if only because he had proved himself more skillful in the game of factional politics.

A one-time protégé of Mr. Kakuei Tanaka, he thrust himself forward when the leader was disgraced on a charge of accepting bribes for sale of Lockheed aircraft to Japan and debilitated by physical ailments. Mr. Takeshita took away most of Mr. Tanaka's following and now leads the biggest faction in the LDP. Mr. Nakasone persuaded Mr. Miyazawa and Mr. Abe to accept Mr. Takeshita's leadership. An election would most probably have led to the same result. Mr. Takeshita seemed to have forged a firm alliance with at least two other factions and put in his bag the votes necessary for a win. How Mr. Takeshita will fare after taking over the reins of Government in 1987 is not so certain.

He will be Japan's first Prime Minister with a humble rural origin. A dichotomy in his nature shows through his record of teaching English in a junior high school and not trying to speak that language in public later. When he was the Minister of Finance, he gave the impression of an extremely cautious man with a reverence for consensus but challengingly titled a book on his ideas 'Going My Way'. Mr. Takeshita says that continuing Mr. Nakasone's programmes would be the basis of his policy. This is not saying enough.

Japan faces two main issues, tax reforms and relations with United States. Mr. Nakasone's plan to impose an indirect tax ran into effective opposition, and the friction with the U.S. over trade continues. Mr. Takeshita cannot be facing an easy future as Japan's next leader and there is nothing to show yet that he will be drawing on secret reserves of dynamism.


Q.The politician who had been Prime Minister for the longest period since the Second World War was


  1. Hirohito
  2. Kakuei Tanaka
  3. Nakasone
  4. Eisaku Sato

Ans.d


Q.When did Hirohito ascend the throne?


  1. 1946
  2. 1926
  3. In the early fifties
  4. 1936

Ans.b


Q.Mr. Tanaka ceased to be Prime Minister because


  1. he could not get a favourable legislative bill passed by Parliament.
  2. he had completed the prescribed two years term.
  3. he was involved in a bribe scandal.
  4. of horse-trading among his party members.

Ans.c


Q.The politician who had just recently ceased to be Prime Minister is


  1. Eisaku Sato
  2. Yasuhiro Nakasone.
  3. Shintaro Abe.
  4. Kiichi Miyazawa.

Ans.b


Q.Mr. Takeshita's success in the Prime Ministerial quest is due to


  1. his financial wizardry.
  2. his loyalty to his predecessor's policies.
  3. his skill in manipulating fractional politics.
  4. his good knowledge of English

Ans.c


Q.The author's assessment of the potential of Mr. Takeshita to be a successful Prime Minister can be summarized as one of


  1. cautious optimism
  2. enthusiastic adulation.
  3. objective skepticism.
  4. undisguised derision.

Ans.c


Q.Factionalism in the Liberal Democratic Party is mainly due to


  1. the clash between urban and rural interests.
  2. the long reign of the Emperor.
  3. fears about one-man leadership
  4. frequent changes in Prime Ministers.

Ans.d


Q.Most of the erstwhile Prime Ministers of Japan


  1. were English educated
  2. were from rural areas.
  3. had urban backgrounds
  4. have been former Finance Ministers.

Ans.c


Q.The number of erstwhile Prime Ministers mentioned by name in the passage is


  1. 2
  2. 3
  3. 4
  4. 5

Ans.b


Reading Comprehensions


The narrator of Midnight's Children describes it as a kind of collective fantasy. I suppose what he, or I, through him was trying to say, was that there never had been a political entity called India until 1947. The thing that became independent had never previously existed, except that there had been an area, a zone called India. So it struck me that what was coming into being, this idea of a nation-state, was an invention. It was an invention of the nationalist movement. And a very successful invention. One could argue that nation-states are a kind of collective fantasies. Very similar things happened with the unification of Italy, and also with the unification of Germany. The history of India is a history of independent nation-states.

It is a history of Oudh or Bengal or Maratha kingdoms. All those independent histories agreed to collectivise themselves into the idea of the nation of India. In the case of Pakistan, it was less successful. Pakistan was under-imagined. It did not survive as a nation-state. If you ask people in general, they would have absolutely no problem with the idea of India at all. I think, in a way the strength of the nationalist idea is shown by its ability to survive the extraordinary stresses that it was placed under. I think the stresses of things — communalism, the high degree of public corruption, of regional rivalries, of the tension between the centre and the state, the external pressures of bad relations with Pakistan — these are colossal pressures which any state could be forgiven for being damaged by.

I think the thing to say about the success of the idea is that it remains an idea though people might not find it very easy to give a simple definition of it. But that it does exist and that it is something to which people feel they belong, I think is now the case. That it survives these stresses is an indication of its strength. I'm not interested in an idealised, romantic vision of India, I know it is the great pitfall of the exile. So you know for me, always, the issue of writing about India has been not to write as an outsider. On the other hand, evidently something has changed in the last 10 years, which is that as a result of various circumstances, I've not been able to return. All I can say is that I have felt it as the most profound loss and I still do.

There have been many losses in the last decade but the loss of the easy return to India has been for me an absolute anguish, an inescapable anguish. I feel as if I've lost a limb. I am very anxious to bring that period to an end. I do not think that one of the most interesting phenomena for India as a country is the phenomenon of the Indian Diaspora. I often think Indian — Indian Indians — find that very hard to understand. In England, when people call themselves British Indian, they mean both halves of that. And yet, what it means to be a British Indian is very alien to an Indian Indian. The same is true in the Caribbean, in Africa, in Canada, in the United States, and so on.

The thing that has interested me is that there are now many, many ways of being something which you can legitimately call Indian. Being an Indian in India is just one of those ways. The forces of disintegration are always there. I think in every society there is the tension between the forces that bring it together and the forces that pull it apart. I'm worried, above all, of the communal issue because half a century is no time at all in the eye of history, and half a century ago something of colossally horrible proportion took place. The fact that it hasn't happened for 50 years on quite the same scale means nothing. It could still happen tomorrow.

One of the things that I remember very vividly, being there 10 years ago at about the time of the killings that took place in Assam, is discussing this with good friends and fellow writers. And I remember somebody said to me, until we understand that we are capable of these things, we can't begin to move beyond them. Because it's a very easy response to atrocities, to say: oh those terrible people did that, and we are not like that. I think the difficult response is to accept we are also capable of that, the thing that happened there could also, in certain circumstances, be something that we were able to perpetrate.

The civilising influence is what prevents most of us from giving vent to those terrible urges. Those urges are part of humanity as well as the more civilized urges. Of course, I fear in India the recurrence of communal or regionalist inter-community violence. I fear the long-term damage to a democracy that can be done by mass corruption. I think corruption is in a way a subversion of democracy and the commonplace view in India is that corruption is everywhere. In a sense, you could say that is not a democratic society. If money, favour and privilege is what makes the place work, then that's not a democracy. At least it runs the danger of being no longer able to call itself a democracy.

What was happening, I thought, was that people were trying to seize control of that rhetoric. That is to say, special interest groups. You could say Hindus are a very large special interest group. If any group inside such a complex and many faceted country tries to define the nation exclusively in its own terms, then it begins to create terrible stresses. I do think that the kind of attempt to define India in Hindu terms is worrying for that reason. It creates backlashes, it creates polarisation, and it creates the risk of more upheaval. Partly, I am saying this as a kind of objective observer, but nobody is an objective observer.

I come from an Indian minority, I no doubt have a minority perspective. I can't ignore that and nor would I wish to. Partly, also I am speaking temperamentally. That is to say, the kind of religious language in politics is something I find temperamentally unpleasant. I don't like people who do that, whether they be sectarians in Northern Ireland or India. I believe in, if possible, separating one's personal spiritual needs and aspirations from the way in which a country is run. I think in those countries where that separation has not taken place, one can see all kinds of distortions of social and ordinary life which are unpleasant.

Iran is an obvious example. The country in which that kind of separation has completely fragmented it. Where Naipaul is right, although I don't share his conclusions about it, but I think where he is right, is in saying that this is a great historical moment. One reason why the 50th anniversary is interesting is that it does seem to represent the end of the first age and the beginning of second age. And to that extent that is true now, if someone was born today, they would be born into a very different set of cultural assumptions and hopes than somebody born 50 years ago.

We were entirely sold on the Nehru-Gandhi kind of plan. We grew up and that was the portrait of the nation we had hung on our wall, and to the extent that you never entirely lose those formative ideas, that's still the picture of the country I've got on my wall. But it's clear that for somebody being born now, they are being born into a very different country. I also think of taking the Naipaul point on what would happen if the BJP were to form a government. Well, what I would like to think is that in order for the BJP or anybody of that persuasion to form a government, they would have to change.

There is even some kind of suggestion that it may even be happening a little bit because they are intelligent people. They understand their weaknesses as well as their strengths. Clearly, for a Hinduist party to form the government of the country is not at all unlikely. So I think one does have to engage with that in the same way as many people in the country who, like myself, were not remotely in tune with the Thatcherite revolution but have to engage with it because it was in fact happening, and kept winning elections, and the world was not going to go back. So, of course, both people inside the Hindu political enterprise and people outside it will have to shift.

I am optimistic about India's ability to force those changes that are necessary because I do believe it is not fundamentally an intolerant country and will not fundamentally accept intolerant politics. On the other hand, there has to be reckoning with the fact that these are ideas, which are gaining in popularity. I'll tell you where I would draw the line myself. I think there was a great historical mistake made in Europe about the Nazi Party. People attempted to see whether they could live with it and discovered very rapidly that was a mistake, that appeasement was a great historical mistake. So, it seems to me, the question is: What do we make of this political enterprise? Is it fundamentally democratic or fundamentally anti-democratic?

If democratic, then we must all learn to make the best of it. If anti-democratic, then we must fight it very hard. What happened in India happened before the book (Satanic Verses) had actually entered. It happened because of an article in India Today, which, I must say, I thought was an irresponsibly written article, because it was written by somebody, who, as a friend, asked me for an early copy of the book, and then presented that book in the most inflammatory sort of way. This was one of the things that disappointed me, that after a lifetime of having written from a certain sensibility, and a certain point of view, I would have expected people in India to know about it since it was all entirely about India. It was written from a deep sense of connection and affection for India. I would have expected that I had some money in the bank. That is to say, if Salman Rushdie wrote any book, then we know who he is.

He is not some idiot who just arrived from nowhere shouting abuse. This is somebody whose work, whose opinions, whose lectures and whose stories we know. I would have hoped that my work would have been judged in the context of what people already knew about me. Instead, it seemed as if everything I had been in my life up to that point suddenly vanished out of the window and this other Rushdie was invented who was this complete bastard who had done this terrible thing. There did not seem to be any attempt to correct that or to combat that. I was surprised and disappointed it did not. It didn't happen here either. It didn't happen anywhere in the world. It was as if the force of history, the force of a historical event was so huge that it erases all that goes before it.

The negative response to the Satanic Verses, let us remember that there was also a positive response, was such that it erased my personality and put in its place some other guy who they didn't recognize at all. Anybody who knows anything about these countries, and I do know something about these countries, knows that every cheap politician can put a demonstration in the street in five minutes. That doesn't represent in any sense the people's will. It represents a certain kind of political structure, political organization. It doesn't represent truth. But I always believed and I still believe that India would come back. I never believe that the loss of India is forever. Because India is not Iran, it's not even Pakistan, and I thought good sense will prevail in India because that's my life experience of Indian people and of the place.


Q.The idea of India that inspired the writer's generation was the one dominated or formed by


  1. the Nehru-Gandhi politics.

  2. the Nehru-Gandhi ideology.

  3. the Nehru-Gandhi idea regarding India's formative years.

  4. the Nehruvian idea of socialism


Ans . b


Q.The writer does not share


  1. Naipaul's stand that the 50th anniversary is a historical moment.

  2. Naipaul's stand that the 50th anniversary is not a historical moment

  3. Naipaul's conclusion on the 50th anniversary being a historical moment.

  4. Naipaul's conclusion on the 50th anniversary not being a historical moment.


Ans . c


Q.The writer shows faith in India's basic


  1. stability

  2. resilience.

  3. fortitude.

  4. democracy.


Ans . b


Q.According to the writer,


  1. politicians incite the general public to demonstrate against writers

  2. a politician's demonstration does not reflect the people's will.

  3. Both (a) and (b)

  4. Neither (a) nor (b)


Ans . c


Q.The writer's view of India is determined mainly by


  1. his experience.

  2. his fondness for the country.

  3. his love for the resilience of the Indian people.

  4. his love of writing about India


Ans . a


Q.According to the writer's friend,


  1. we should fight against communal pressure

  2. the fact that communal conflagrations haven't occurred in India for half a century, is something to be proud of.

  3. we can move beyond things, only after we know we are capable of those things.

  4. we have to identify with the people who were involved in inciting communal passions.


Ans . c


Q.What according to the passage prevents us from giving in to violent, terrible urges?


  1. Our education

  2. Our upbringing

  3. Our cultural influences

  4. The civilising influence


Ans . d


Q.According to the writer, what disqualifies India from being called a democracy?


  1. Its communalism

  2. Its corruption

  3. Its anti-minority stance

  4. All of these


Ans . b


Q.The writer contradicts his assertion of being an 'objective observer' on the basis that


  1. no one can be an 'objective observer'.

  2. no one is an 'objective observer'.

  3. he is a subjective observer.

  4. everybody is a subjective observer


Ans . b


Q.In the first paragraph of the passage, the writer questions


  1. the existence of a political entity called India prior to independence.

  2. the contention that a political entity called India did not exist.

  3. the stand that India was an invention.

  4. the stand that India needs to think in terms of its being a nation-state.


Ans . a


Q.According to the writer, the difference between India and Pakistan was that


  1. India survived as a nation-state, Pakistan did not.

  2. Indians were full of fantastic ideas in 1947

  3. Pakistan was born out of another nation.

  4. the creation of Pakistan suffered from under-imaginativeness.


Ans . d


Q.According to the passage, the secret of India's survival lies in


  1. its ability to fight back in the face of tremendous stress and strains

  2. the highly fertile imagination of the Indian people.

  3. a sense of belonging that people feel for it.

  4. Both (a) and (c)


Ans . d


Reading Comprehensions


If Western civilization is in a state of permanent crisis, it is not far-fetched to suggest that there may be something wrong with its education. No civilization, I am sure, has ever devoted more energy and resources to organised education, and if we believe in nothing else, we certainly believe that education is, or should be, the key to everything. In fact, the belief in education is so strong that we treat it as the residual legatee of all our problems. If the nuclear age brings new danger; if the advance of genetics engineering opens the doors of new abuses; if commercialism brings new temptations, the answer must be more and better education. The modern way of life is becoming more complex:

this means that everybody must become more highly educated. “By 1984,” it was said recently, “it will be desirable that the most ordinary of men is not embarrassed by the use of a logarithm table, the elementary concepts of the calculus, and by the definitions and uses of such words as electron, coulomb, and volt. He should further have become able not only to handle a pen, and ruler but also a magnetic tape, valve, and transistor. The improvement of communications between individuals and groups depends on it.” Most of all, it appears, the international situation calls for prodigious educational efforts. The classical statement on this point was delivered by Sir Charles (now Lord) Snow in his Rede Lecture some years ago: To say that we must educate ourselves or perish, is a little more melodramatic than the facts warrant.

To say we have to educate ourselves or watch a steep decline in our lifetime, is about right. According to Lord Snow, the Russians are apparently doing much better than anyone else and will 'have a clear edge', unless and until the Americans and we educate ourselves both sensibly and imaginatively'. Lord Snow, it will be recalled, talked about 'The Two Cultures and the Scientific Revolution' and expressed his concern that 'the intellectuals life of the whole of western society is increasingly being split into two polar groups . . . At one pole we have the literary intellectuals . . . at the other the scientists'. He deplores the 'gulf of mutual incomprehension' between these two groups and wants it bridged. It is quite clear how he thinks this 'bridging' operation is to be done; the aims of his educational policy would be, first, to get as many 'alpha-plus scientists as the country can throw up';

second, to train 'a much larger stratum of alpha professionals' to do the supporting research, high class design and development; third, to train 'thousands upon thousands' of other scientists and engineers; and finally, to train 'politicians, administrators, and entire community, who know enough science to have a sense of what the scientists are talking about'. If this fourth and last group can at least be educated enough to 'have sense' of what the real people, the scientists and engineers, are talking about, so Lord Snow seems to suggest, the gulf of mutual incomprehension between the 'Two Cultures' may be bridged.

These ideas on education, which are by no means unrepresentative of our times, leave one with the uncomfortable feeling that ordinary people, including politicians, administrators, and so forth, are really not much use, they have failed to make the grade: but, at least, they should be educated enough to have a sense of what is going on, and to know what the scientists mean when they talk to quote Lord Snow's example about the Second Law of Thermodynamics. It is an uncomfortable feeling, because the scientists never tire of telling us that the fruits of their labours are 'neutral': whether they enrich humanity or destroy it depends on how they are used. And who is to decide how they are used? There is nothing in the training of scientists and engineers to enable them to take such decision, or else, what becomes of the neutrality of science?

If so much reliance is today being placed in the power of education to enable ordinary people to cope with the problems thrown up by scientific and technological progress, then there must be something more to education than Lord Snow suggests. Can education help us to turn the potentiality into a reality to the benefit of man? To do so, the task of education would be, first and foremost the transmission of ideas of value, of what to do with our lives. There is no doubt also the need to transmit know-how but this must take second place, for it is obviously somewhat foolhardy to put great powers into the hands of people without making sure that they have a reasonable idea of what to do with them.

At present, there can be little doubt that the whole of mankind is in mortal danger, not because we are short of scientific and technological know-how, but because we tend to use it destructively, without wisdom. More education can help us only if it produces more wisdom. The essence of education, I suggested, is the transmission of values, but values do not help us to pick our way through life unless they have become our own, a part, so to say, of our mental make-up. This means that they are more than mere formulae or dogmatic assertions: that we think and feel with them, that they are the very instruments through which we like and interpret, and experience the world. When we think, we do not just think: we think with ideas. Our mind is not a blank, a tabula rasa. When we begin to think we can do so only because our mind is already filled with all sorts of ideas with which to think.

All through our youth and adolescence, before the conscious and critical mind begins to act as a sort of censor and guardian at the threshold, ideas seep into our mind, multitudes of them. These years are, one might say, our Dark Ages during which we are nothing but inheritors; it is only in later years that we can gradually learn to sort out our inheritance. First of all, there is language. Each word is an idea. If the language which seeps into us during our Dark Ages is English, our mind is thereby furnished by a set of ideas which is significantly different from the set represented by Chinese, Russian, German, or even American. Apart from words there are the rules of putting them together: grammar, another bundle of ideas, the study of which has fascinated some modern philosophers to such an extent that they thought they could reduce the whole of philosophy to a study of grammar. All philosophers and others have always paid a great deal of attention to ideas seen as the result of thought and observation;

but in modern times all too little attention has been paid to the study of the ideas which form the very instruments by which thought and observation proceed. On the basis of experience and conscious thought small ideas may easily be dislodged, but when it comes to bigger, more universal, or more subtle ideas, it may not be so easy to change them. Indeed, it is often difficult to become aware of them, as they are the instruments and not the result of our thinking just as you can see what is outside you, but cannot easily see that with which you see, the eye itself. And even when one has become aware of them it is often impossible to judge them on the basis of ordinary experience. We often notice the existence of more or less fixed ideas in other people's minds — ideas with which they think without being aware of doing so.

We then call them prejudices, which is logically quite correct because they have merely seeped into the mind and are in no way the result of judgement. But the word prejudice is generally applied to ideas that are patently erroneous and recognisable as such by anyone except the prejudiced man. Most of the ideas with which we think are not of that kind at all. To some of them, like those incorporated in words and grammar, the notions of truth or error cannot even be applied, others are quite definitely not prejudices but the result of a judgement; others again are tacit assumptions or presuppositions which may be very difficult to recognise.

I say, therefore, that we think with or through ideas and that what we call thinking is generally the application of pre-existing ideas to a given situation or set of facts. When we think about, say the political situation we apply to that situation our political ideas, more or less systematically, and attempt to make that situation' 'intelligible' to ourselves by means of these ideas. Similarly, everywhere else we evaluate the situation in the light of our value-ideas. The way in which we experience and interpret the world obviously depends very much indeed on the kind of ideas that fill our minds.

If they are mainly small, weak, superficial, and incoherent, life will appear insipid, uninteresting, petty and chaotic. It is difficult to bear the resultant feeling of emptiness, and the vacuum of our minds may only too easily be filled by some big, fantastic notion-political or otherwise — which suddenly seem to illumine everything and to give meaning and purpose to our existence. We feel that education will help solve each new problem or complexity that arises. It needs no emphasis that herein lies one of the great dangers of our times.

When people ask for education they normally mean something more than mere training, something more than mere knowledge of facts, and something more than a mere diversion. Maybe they cannot themselves formulate precisely what they are looking for; but I think what they are really looking for is ideas that could make the world, and their own lives, intelligible to them. When a thing is intelligible you have a sense of participation; when a thing is unintelligible you have a sense of estrangement.

'Well, I don't know', you hear people say, as an impotent protest against the unintelligibility of the world as they meet it. If the mind cannot bring to the world a set — or, shall we say, a tool box — of powerful ideas, the world must appear to it as a chaos, a mass of unrelated phenomena, of meaningless events. Such a man is like a person in a strange world and without any signs of civilization, without maps or signposts or indicators of any kind. Nothing has any meaning to him; nothing can hold his vital interest; he has no means of making anything intelligible to himself.


Q.The writer's contention in the passage is that the crisis in Western civilization can be explained by


  1. the presence of some flaws in its education

  2. some inherent lack of coordination among its various elements

  3. some basic misunderstanding in its society.

  4. the energy it has devoted to education


Ans . A


Q.According to the writer, Lord Snow sees the intellectual life of Western society as split between


  1. the educated and the uneducated

  2. the government servants and the plebeians.

  3. scientists and literary intellectuals.

  4. administrators and intellectuals.


Ans . C


Q.The writer seems to criticise the belief that


  1. education gives rise to further complexities as civilization progresses.

  2. all new problems and complexities can be tackled and solved by more and better education.

  3. people need to learn more in order to earn more

  4. None of these


Ans . B


Q.What, according to the author, would be the definition of 'prejudice'?


  1. Ideas that help people to identify with new situations

  2. Fixed ideas with which people think without being aware of doing so

  3. Ideas that people cull from experience in order to judge a situation.

  4. Fixed ideas that see a person through the trials and tribulations of life.


Ans . B


Q.According to Lord Snow, which of the following groups needs to be educated enough to at least understand the works of scientists and engineers?


  1. Politicians, administrators, and the entire community

  2. Politicians and literary intellectuals

  3. Politicians and the layman

  4. All of these


Ans . A


Q.In the passage, the writer questions


  1. the neutrality of science.

  2. scientists' stand on the neutrality of science.

  3. scientists' stand on the neutrality of their labours.

  4. Lord Snow's assertion regarding the potential of intellectuals in society.


Ans . C


Q.The author's assertion in the passage is that education's main responsibility is to


  1. transmit ideas of value

  2. transmit technical knowledge.

  3. Both (a) and (b)

  4. transmit the values regarding human and societal norms.


Ans . A


Q.The author believes that


  1. the gulf between science and literature needs to be bridged.

  2. ideas should be maintained for a holistic view of society and its problems.

  3. words are not ideas.

  4. None of these


Ans . D


Q.Which of the following sentences is not true according to the author?


  1. Values must be part of one's psyche.

  2. Values are merely dogmatic assertions.

  3. One identifies with values.

  4. Values are the means to interpret and experience the world.


Ans . B


Q.Thinking is


  1. being.

  2. knowing.

  3. application of pre-existing ideas to a situation

  4. application of fixed ideas to a situation.


Ans . C


READING COMPREHENSIONS


The highest priced words are ghost-written by gagmen who furnish the raw material for comedy over the air and on the screen. They have a word-lore all their own, which they practise for five to fifteen hundred dollars a week, or fifteen dollars a gag at piece rates. That's sizable rate for confounding acrimony with matrimony, or extracting attar of roses from the otter. Quite apart from the dollar sign on it, gagmen's word-lore is worth a close look, if you are given to the popular American pastime of playing with words — or if you're part of the 40 per cent who make their living in the word trade.

Gag writers' tricks with words point up the fact that we have two distinct levels of language: familiar, ordinary words that everybody knows; and more elaborate words that don't turn up so often, but many of which we need to know if we are to feel at home in listening and reading today. To be sure gagmen play hob with the big words, making not sense but fun of them. They keep on confusing bigotry with bigamy, illiterate with illegitimate, monotony with monogamy, osculation with oscillation. They trade on the fact that for many of their listeners, these fancy terms linger in a twilight zone of meaning. It's their deliberate intent to make everybody feel cozy at hearing big words, jumbled up or smacked down.

After all, such words loom up over-size in ordinary talk, so no wonder they get the bulldozer treatment from the gagmen. Their wrecking technique incidentally reveals our language as full of tricky words, some with 19 different meanings, others which sound alike but differ in sense. To ring good punning changes, gag writers have to know their way around in the language. They don't get paid for ignorance, only for simulating it. Their trade is a hard one, and they regard it as serious business. They never laugh at each other's jokes; rarely at their own. Like comediennes, they are usually melancholy men in private life. Fertile invention and ingenious fancy are required to clean up 'blue' burlesque gags for radio use. These shady gags are theoretically taboo on the air.

However, a gag writer who can leave a faint trace of bluing when he launders the joke is all the more admired — and more highly paid. A gag that keeps the blue tinge is called a 'double intender', gag-land jargon for double entendre. The double meaning makes the joke funny at two levels. Children and other innocents hearing the crack for the first time take it literally, laughing at the surface humour; listeners who remember the original as they heard it in vaudeville or burlesque, laugh at the artfulness with which the blue tinge is disguised. Another name for a double meaning of this sort is 'insinuendo'. This is a portmanteau word or 'combo', as the gagmen would label it, thus abbreviating combination.

By telescoping insinuation and innuendo, they get insinuendo, on the principle of blend words brought into vogue by Lewis Caroll. 'Shock logic' is another favourite with gag writers. Supposedly a speciality of women comediennes, it is illogical logic more easily illustrated than defined. A high school girl has to turn down a boy's proposal, she writes: Dear Jerry, I'm sorry, but I can't get engaged to you. My mother thinks I am too young to be engaged and besides, I'm already engaged to another boy. Yours regretfully. Guess who. Gag writers' lingo is consistently funnier than their gags. It should interest the slang-fancier.

And like much vivid jargon developed in specialised trades and sports, a few of the terms are making their way into general use. Gimmick, for instance, in the sense either of a trick devised or the point of a joke, is creeping into the vocabulary of columnists and feature writers. Even apart from the trade lingo, gagmen's manoeuvres are of real concern to anyone who follows words with a fully awakened interest. For the very fact that gag writers often use a long and unusual word as the hinge of a joke, or as a peg for situation comedy, tells us something quite significant: they are well aware of the limitations of the average vocabulary and are quite willing to cash in on its shortcomings. When Fred Allens' joke-smiths work out a fishing routine, they have Allen referring to the bait in his most arch and solemn tones:

"I presume you mean the legless invertebrate." This is the old minstrel trick, using a long fancy term, instead of calling a worm a worm. Chico Marx can stretch a pun over 500 feet of film, making it funnier all the time, as he did when he rendered, "Why a duck?" And even the high-brow radio writers have taken advantage of gagmen's technique. You might never expect to hear on the air such words as lepidopterist and entymologist. Both occur in a very famous radio play by Norman Corvine, 'My client Curly', about an unusual caterpillar which would dance to the tune 'yes, sir, she's my baby' but remained inert to all other music.

The dancing caterpillar was given a real New York buildup, which involved calling in the experts on butterflies and insects which travel under the learned names above. Corvine made mild fun of the fancy professional titles, at the same time explaining them unobtrusively. There are many similar occasions where any one working with words can turn gagmen's trade secrets to account. Just what words do they think outside the familiar range? How do they pick the words that they 'kick around'? It is not hard to find out.


Q.According to the writer, a larger part of the American population


  1. indulges in playing out the role of gag writers.

  2. indulges in the word trade.

  3. seeks employment in the gag trade for want of something better.

  4. looks down on gag writers.


Ans . B


Q.The hallmark of gag writers is that they


  1. ruin good, simple language.

  2. have fun with words.

  3. make better sense of words

  4. play with words to suit the audience's requirements


Ans . B


Q.According to the passage, the second level of language is important if


  1. one wants to feel at home reading and listening today.

  2. one wants to be a gag writer.

  3. one wants to understand clean entertainment.

  4. All of these


Ans . A


Q.According to the writer, gag writers thrive on


  1. the double-layered aspect of language.

  2. audience craze for double entendres

  3. vulgar innuendoes

  4. commonplace jugglery with language


Ans . A


Q.In gag writers' trade


  1. long words are abbreviated for effect.

  2. parts of words are combined to produce a hilarious portmanteau effect.

  3. long words play a major role

  4. Both (b) and (c)


Ans . D


Q.When the writer says, "They don't get paid for ignorance, only for simulating it," he means to say


  1. the audience likes to think the gag writers are an ignorant lot.

  2. gag writers are terrific with insinuations.

  3. simulating ignorance is the trick that makes gag writers tick.

  4. None of these


Ans . C


Q.Gag writers have influenced


  1. television artistes

  2. radio writers.

  3. circus clowns.

  4. All of these


Ans . B


Reading Comprehensions


From ancient times, men have believed that, under certain peculiar circumstances, life could arise spontaneously: from the ooze of rivers could come eels and from the entrails of dead bulls, bees; worms from mud, and maggots from dead meat. This belief was held by Aristotle, Newton and Descartes, among many others, and apparently the great William Harvey too. The weight of centuries gradually disintegrated men's beliefs in the spontaneous origin of maggots and mice, but the doctrine of spontaneous generation clung tenaciously to the question of bacterial origin.

In association with Buffon, the Irish Jesuit priest John Needham declared that he could bring about at will the creation of living microbes in heat-sterilised broths, and presumably, in propitiation, theorised that God did not create living things directly but bade the earth and water to bring them forth. In his Dictionaire Philosophique, Voltaire reflected that it was odd to read of Father Needham's claim while atheists conversely should deny a Creator yet attribute to themselves the power of creating eels. But, wrote Thomas Huxley, 'The great tragedy of science — the slaying of a beautiful hypothesis by an ugly fact — which is so constantly being enacted under the eyes of philosophers, was played, almost immediately, for the benefit of Buffon and Needham. The Italian Abbé Spallanzani did an experiment.

He showed that a broth sealed from the air while boiling never develops bacterial growths and hence never decomposes. To Needham's objection that Spallanzani had ruined his broths and the air above them by excessive boiling, the Abbé replied by breaking the seals of his flasks. Air rushed in and bacterial growth began! But the essential conflict remained. Whatever Spallanzani and his followers did to remove seeds and contaminants was regarded by the spontaneous generationists as damaging to the 'vital force' from whence comes new life. Thus, doubt remained, and into the controversy came the Titanic figure of Louis Pasteur.

Believing that a solution to this problem was essential to the development of his theories concerning the role of bacteria in nature, Pasteur freely acknowledged the possibility that living bacteria very well might be arising anew from inanimate matter. To him, the research problem was largely a technical one: to repeat the work of those who claimed to have observed bacterial entry. For the one that contended that life did not enter from the outside, the proof had to go to the question of possible contamination. Pasteur worked logically. He found during the experiments that after prolonged boiling, a broth would ferment only when air was admitted to it.

Therefore, he contended, either air contained a factor necessary for the spontaneous generation of life or viable germs were borne in by the air and seeded in the sterile nutrient broth. Pasteur designed ingenious flasks whose long S-shaped necks could be left open. Air was trapped in the sinuous glass tube. Broths boiled in these flask tubes remained sterile. When their necks were snapped to admit ordinary air, bacterial growth would then commence — but not in every case. An occasional flask would remain sterile presumably because the bacterial population of the air is unevenly distributed. The forces of spontaneous generation would not be so erratic.

Continuous scepticism drove Pasteur almost to fanatical efforts to control the ingredients of his experiments to destroy the doubts of the most sceptical. He ranged from the mountain air of Montanvert, which he showed to be almost sterile, to those deep, clear wells whose waters had been rendered germfree by slow filtration through sandy soil.

The latter discovery led to the familiar porcelain filters of the bacteriology laboratory. With pores small enough to exclude bacteria, solutions allowed to percolate through them could be reliably sterilised. The argument raged on and soon spilled beyond the boundaries of science to become a burning religious and philosophical question of the day. For many, Pasteur's conclusions caused conflict because they seemed simultaneously to support the Biblical account of creation while denying a variety of other philosophical systems.

The public was soon caught up in the crossfire of a vigorous series of public lectures and demonstrations by leading exponents of both views, novelists, clergymen, their adjuncts and friends. Perhaps the most famous of these evenings in the theatre — competing perhaps with a great debate between Huxley and Bishop Wiberforce for elegance of rhetoric — was Pasteur's public lecture at the Sorbonne on April 7, 1864. Having shown his audience the swan necked flasks containing sterile broths, he concluded, "And, therefore, gentlemen, I could point to that liquid and say to you, I have taken my drop of water from the immensity of creation, and I have taken it full of the elements appropriated to the development of inferior beings.

And I wait, I watch, I question it! — begging it to recommence for me the beautiful spectacle of the first creation. But it is dumb, dumb since these experiments were begun several years ago; It is dumb because I have kept it from the only thing man does not know how to produce: from the germs that float in the air, from life, for life is a germ and a germ is life. Never will the doctrine of spontaneous generation recover from the mortal blow of this simple experiment." And it is not. Today these same flasks stand immutable: they are still free of microbial life. It is an interesting fact that despite the ringing declaration of Pasteur, the issue did not die completely.

And although far from healthy, it is not yet dead. In his fascinating biography of Pasteur, Rene Dubos has traced the later developments which saw new eruptions of the controversy, new technical progress and criticism, and new energetic figures in the breach of the battle such as Bastion, for, and the immortal Tyndall, against, the doctrine of spontaneous generation. There was also new 'sorrow' for Pasteur as he read years later, in 1877, the last jottings of the great physiologist Claude Bernard and saw in them the 'mystical' suggestion that yeast may arise from grape juice. Even at this late date, Pasteur was stirred to new experiments again to prove to the dead Bernard and his followers the correctness of his position.

It seems to me that spontaneous generation is not only a possibility, but a completely reasonable possibility which should never be relinquished from scientific thought. Before men knew of bacteria, they accepted the doctrine of spontaneous generation as the 'only reasonable alternative' to a belief in supernatural creation. But today, as we look for satisfaction at the downfall of the spontaneous generation hypothesis, we must not forget that science has rationally concluded that life once did originate on earth by spontaneous generation. It was really Pasteur's evidence against spontaneous generation that for the first time brought the whole difficult question of the origin of life before the scientific world.

In the above controversy, what was unreasonable was the parade of men who claimed to have 'proved' or who resolutely 'believed in' spontaneous generation on the face of proof — not that spontaneous generation cannot occur — but that their work was shot through with experimental error. The acceptable evidence also makes it clear that spontaneous generation, if it does not occur, must obviously be a highly improbable event under present conditions. Logic tells us that science can only prove an event improbable: it can never prove it impossible — and Gamow has appropriately remarked that nobody is really certain what would happen if a hermetically sealed can were opened after a couple of million years.

Modern science agrees that it was highly improbable for life to have arisen in the pre-Cambrian seas, but it concluded, nevertheless, that there it did occur. With this, I think, Pasteur would agree. Aside from their theoretical implications, these researchers had the great practical result of putting bacteriology on a solid footing. It was now clear how precisely careful one had to be to avoid bacterial contamination in the laboratory. We now knew what 'sterile' meant and we knew that there could be no such thing as 'partial sterilization'. The discovery of bacteria high in the upper atmosphere, in the mud of the deep sea bottom, in the waters of hot springs, and in the Arctic glaciers established bacterial ubiquity as almost absolute.

In recognition of this Lord Lister introduced aseptic technique into the practice of surgery. It was the revolution in technique alone that made possible modern bacteriology and the subsequent research connecting bacteria to phenomena of human concern, research, which today is more prodigious than ever. We are just beginning to understand the relationship of bacteria to certain human diseases, to soil chemistry, nutrition, and the phenomenon of antibiosis, wherein a product of one organism (e.g. penicillin) is detrimental to another. It is not an exaggeration then to say that the emergence of the cell theory represents biology's most significant and fruitful advance.

The realisation that all plants and animals are composed of cells which are essentially alike, that cells are all formed by the same fundamental division process, that the total organism is made up of activities and inter-relations of its individual cells, opened up horizons we have not even begun to approach. The cell is a microcosm of life, for in its origin, nature and continuity resides the entire problem of biology.


Q. Needham's theory that 'God did not create living things directly' was posited as


  1. an attempt to support his assertion by religious doctrine

  2. an attempt to placate his religious peers.

  3. an attempt at propitiating a possibly offended God or the religious psyche of the time.

  4. All of these


Ans . C


Q. It can be inferred from the passage that


  1. Huxley, Buffon and Needham were contemporaries.

  2. Buffon, Needham, Voltaire and Huxley were contemporaries

  3. Voltaire wrote a treatise on Needham's claim.

  4. None of these


Ans . B


Q. According to the passage,


  1. Pasteur's precursors in the field worked on the basis of spontaneous generation.

  2. unlike his predecessors, Pasteur worked on logical premises rather than arbitrary and spontaneous discoveries

  3. Pasteur stood to benefit largely from the work of his predecessors.

  4. Pasteur developed the ideas set forth by Voltaire and Needham


Ans . B


Q. Pasteur began his work on the basis of the contention that


  1. either air contained a factor necessary for the spontaneous generation of life or viable germs were borne in by the air and seeded in the sterile nutrient broth.

  2. after prolonged boiling, a broth would ferment only when air was admitted to it.

  3. Both (a) and (b)

  4. Neither (a) nor (b)


Ans . A


Q. The porcelain filters of the bacteriology laboratories owed their descent to


  1. Pasteur's homeland

  2. the well water of Montanvert that had been rendered germ-free by slow filtration through sandy oil.

  3. Both (a) and (b)

  4. None of these


Ans . B


Q. What according to the passage was Pasteur's declaration to the world?


  1. Nobody could deny the work done by him.

  2. Science would forever be indebted to his experiments in bacteriology.

  3. The doctrine of spontaneous generation would never recover from the mortal blow dealt to it by his experiments.

  4. Those who refused to acknowledge his experiments would regret their scepticism


Ans . C


Q. What according to the writer, was the problem with the proponents of spontaneous generation


  1. Their work had no scientific basis.

  2. Their work was ruined by experimental errors.

  3. Both (a) and (b)

  4. Neither (a) nor (b)


Ans . B


Q. One of the results of the theoretical cross fire regarding bacteriology was that


  1. partial sterilization as a possibility was ruled out.

  2. aseptic technique was introduced in surgery

  3. the meaning of sterile was clear to all.

  4. All of these


Ans . A


Q. One of the reasons for the conflict caused by Pasteur's experiments was that


  1. they denied the existence of God as the creator.

  2. they seemed simultaneously to support the Biblical account of creation while denying a variety of other philosophical systems

  3. academicians and scientists refused to accept his theories

  4. there were too many debates on the topic and this left the people confused.


Ans . B


Q. According to the author,


  1. it is an exaggeration to say that cell theory represents biology's most significant and fruitful advance.

  2. Pasteur could not hold his own against the contenders.

  3. cell theory rendered null and void all the other bacteriological theories of the time.

  4. the emergence of the cell theory represents biology's most significant and fruitful advance.


Ans . D


Reading Comprehensions


The end of mutual funds, when it came, was sudden but not unexpected. For over 10 years, mutual fund has been scripting its own growth demise, embarking on a reckless course of high risks, unhealthy pastimes, and unchecked maladies. Ironically but fittingly too, the very hand that had supported and sustained it through the turbulent early period of its existence was the one that, finally wielded the euthanasian syringe. The individual investor it was who had made the mutual fund post-liberalisation, India's most vibrant vehicle for individual investment.

The individual investor it was who brought the curtain down on an act that had started with a virtuoso performance, only to putrefy into a show of ineptitude, imprudence, and irresponsibility. The mutual fund, as we know it, may be dead. It died of many things. But, primarily, of a cancer that ate away at its innards. A cancer that destroyed the value of the investments, the mutual funds was made to service the Rs. 85,000 crore that India's investors had entrusted them with ever since they began life way back in 1964 as The Unit Trust Of India's (UTI), now disgraced Unit Scheme 64(US 64). A cancer that grew from the refusal of the men and women to manage the mutual fund to exercise a mixture of caution and aggression, but to adopt, instead, an indisciplined, unplanned, fire-from-the hip approach to investment.

A cancer that ultimately, robbed the mutual funds of the resources they would have to use to pay back their investors, leaving them on Death Row. Indeed, the scandal that US 64 had been brewing for years, was only one, but not the first, of the warningbells that pointed to the near emptiness of many a mutual fund's coffers. In quick succession have emerged reports of more and more fund-schemes that have been laid bare, their corpuses empty, their ability to meet their promises of assured returns to investors demolished.

At least 37 per cent of the 235 fund schemes in operation in the country have promised investors assured returns of over 15 per cent for 5 years, and repurchase-prices well above their Net Asset Values (NAVs). According to a study conducted by the Delhi-based Value Research, at least 18 big schemes due for redemption over the next three years will be unable to service their investors, or even return their money at the time of redemption. The shortfall? Rs. 4,685.10 crore. Or 75.87 per cent of the amount handed over by trusting investors to fund managers. Worries Ajai Kaul, 38, president, Alliance Capital Asset Management: "When an assured-returns scheme runs into problems, investors view it as one more let-down by the mutual funds."

Had they but known of the actual practices seen in the offices and hallways of the mutual funds, which have translated into these results, investors would have shown their disgust long ago. Take the case of a mutual fund company that manages more than a dozen schemes. According to an unwritten, but formalised, principle, each scheme takes it in turn to sell some of its holdings to its sister-schemes, booking fat notional gains and posting NAVs. While investors responded by pouring in even more of their savings, the profits were clearly only on paper. In the offices of another asset management company half way across Mumbai, the demand for cellular-phones peaked six months ago.

Its employees had, suddenly, realised that making their personal deals using information gathered in the course of their professional work, was best done over cell phones so that the company's records wouldn't show the call being made. Obviously, the hot tips went to fatten their — and not investors' — pockets. Earlier, quite a few merchant bankers entered the mutual funds industry to use the corpus to subscribe to the issues they were managing. It took a crash in the primary market — not ethics or investigations — for this practice to stop. Filled with fear and loathing — and righteous anger — the investor has, therefore, decided to adjure the mutual fund.

According to Marketing And Development Research Associates (MDRA) opinion poll of 342 investors conducted last fortnight in the five metros — Bangalore, Kolkata, Chennai, Delhi and Mumbai — mutual funds as an investment instrument now ranks a lowly fourth on safety — after bank deposits, gold, and real estate — and fifth on returns — ahead only of bank deposits and gold. And only 14.20 per cent of the sample will even consider investing in a mutual fund in the future. Still, it is the species that has died, not its every member. The ones that have survived are the bright performers who beat the market benchmark — the 100 — scrip. The Bombay Stock Exchange (BSE) National Index — by the widest margins within their three genres: growth, income and balance.

However, even their star turns have not been able to stave off the stench of death over the business. In fact, an autopsy of the late — and, at the moment not particularly lamented — mutual funds reveal a sordid saga of callousness and calumny. Sheer disaster stares the mutual funds in the face and a cataclysm could destroy the savings of lakhs of investors too. A Value Research estimate of probable shortfall that 18 assured-returns schemes will face at the time of their scheduled redemptions over the three years adds up to a sense-numbing Rs. 4,685 crore.

An independent audit of the 60 assured-returns schemes managed by the public sector mutual funds conducted by Price Waterhouse Coopers at the behest of the Securities and Exchange Board of India (SEBI) estimated a shortfall of between Rs. 2,500 crore and Rs. 3,000 crore. In 1999 alone judging from their present NAVs, the four schemes due for redemption — Canbank Asset Management Company's Cantriple, IndBank Asset Management Company's IndPrakash, SBI Funds Management's Magnum Triple Plus, and BOI Mutual Fund's (BOIMF) Double Square Plus — are heading for a collective shortfall of Rs. 1,639.55 crore. As of June 30, 1998, the country's 252 fund-schemes managed assets with a market value of Rs. 69,599 crore, with the UTI alone controlling the fate of Rs. 50,000 crore.

That is Rs. 11,000 crore less than the money invested in these schemes as of June 30, 1997, which means that the mutual funds have wiped out Rs. 11,000 crore from the investors' hard earned money in the intervening 12 months. Of course, every fund is paying for the sins of the black sheep. For, the villain of the piece was UTI and the 95 funds managed by the public sector banks and institutions, the value of whose corpuses fell from Rs. 66,748 crore to Rs. 57,350 crore in the past year. In fact, these funds contributed 85.40 per cent of the overall value-loss, with the private sector funds boosting their corpuses from Rs. 4,000 crore to Rs. 4,120 crore to lower the For investors, that has translated into an option of either exiting at a loss — or holding on in vain hope.

On November 20, 1998, a depressing 77 per cent of the 58 listed fund schemes were quoting at discounts of between 5 per cent and 40 per cent to their NAVs. And what of the NAVs themselves? The units of a shoulder-slumping 15 per cent of the schemes were worth less than their par values. And US 64, of course continued to languish, with an estimated NAV of Rs. 9.68. Even if there are schemes that have performed individually well, that the mutual funds have collectively failed to deliver couldn't be more obvious. So investors' murderous mood can hardly be debated. Their genesis and growth reveals just what blinded the mutual funds to the possibility of failure.

Forty per cent of the banks-and-insurance companies-promoted funds in operation were launched between 1987 and 1993, when the stock markets were bull-dominated. In a period that saw only one bear phase, the BSE Sensitivity Index (the Sensex) climbed by 346 per cent. Being successful with equity investments required no skills; only investible funds. Nor was fund-raising a problem, as investors desperately sought ways to grab a piece of equity boom. Between 1984 and 1989, the mutual funds collected Rs. 13,455 crore as subscriptions, but, in the next five years, they picked up Rs. 45,573 crore.

In January, 1994, the UTI's Mastergain mopped up a stunning Rs. 4,700 crore while the most awaited Morgan Stanley Growth — a showcase for the fabled fund-management metier of the foreign mutual funds — took in Rs. 1,000 crore in just three days.

Low entry-barriers — a so called sound track-record, a general reputation of fairness and integrity, an application-fee of Rs. 25,000, a registration fee of Rs. 25 lakh and an annual fee of Rs. 2.50 lakh — made entering the business a snap. Explains Ajay Srinivasan, 34, CEO, Prudential ICICI Mutual Fund: "Mutual funds were misunderstood by investors. Everyone thought they were a one way ticket to a jackpot." Intoxicated, fund-managers poured in more and more of their corpuses into equity, ignoring the downsides, confident that the boom would last forever. In the process, they ignored the very concept of risk-management, blithely ignoring the safety net of fixed-income instruments, and accusing those who advised caution of being cowards.

In 1995, for instance, ABN estimated 70 per cent of the money being managed by the mutual funds had been funnelled into equity. Whether they knew it or not, they were breaking away from the trend set by the mutual funds in the US, where the industry began by investing primarily in the money market, with only 25 per cent of their corpus set aside for stocks. Only in the past 15 years, after operating for more than seven decades, have those funds ventured into equity. Unfortunately, their success blinded the fund-managers to the fact that they were riding a wave-not navigating the treacherous seas.

As Vivek Reddy, 36, CEO, Kothari-Pioneer Mutual Fund, puts it: "It was the stock market conditions that helped the mutual funds deliver returns, not superior investment skills." Then, the stock markets collapsed and never quite recovered. Between July 1997 and October 1998, the Sensex free-fell from 4306 to 2812 finally nullifying the theory that if you wait long enough, share-prices are always bound to rise. And the mutual fund, unused to a diet of falling equity indices, collapsed too. The quantum of money mopped up by the mutual fund may suggest that the reports of its extinction have been greatly exaggerated. In 1997-98, Indians entrusted Rs. 18,701 crore to the mutual funds, with new schemes alone mopping up Rs. 12,279 crore.

Questions R. G Sharma, 58, CEO, LIC Mutual Fund: "How do you explain that Dhanvarsha 12 and Dhanvarsha 13, floated in April and September 1998, managed to mop up Rs. 335 crore?" Not quite a loss of faith, would you say? Think again. In those 12 months, those very investors also took away Rs. 16,227 crore in the form of repurchases and redemptions, leaving only Rs. 2,474 crore more in the hands of fund-managers. What's more, since none of the withdrawals could have been made from the new schemes, the old schemes, obviously, gave it all up, effectively yielding Rs. 9,805 crore to angry investors who took away their money. It is the same story this year: in the first quarter of 1998-99, old schemes collected Rs. 2,340 crore, compared to the new schemes' Rs. 1,735 crore but they gave up Rs. 2,749 crore ending up Rs. 409 crore poorer. Sure, some people are still putting money into the mutual funds.

The real reason: money is flowing in from two genres of investors — neither of whom is the quintessential urban. The first comprises people in the semi-urban and rural areas, for whom names like the LIC and GIC still represent safety and assured schemes of income. Importantly, this category investor isn't clued into the financial markets, and is not, accordingly, aware of the problems that confront the mutual funds. Confirms Nikhil Khatau, 38, Managing Director, Sun F & C Asset Management: "That market is fairly stable. "However, as soon as the fundamental problems hit their dividend-paying ability, even the die hard mutual fund investor from India's villages and small towns — who, don't forget, has already been singed by the disappearance of thousands of nonbanking finance companies — will swear off their favourite investment vehicle.

The second genre of investor explains why the private sector funds have been successful in soaking up large sums: 31.10 per cent of the total takings in 1997-98, and 10.70 per cent in the first quarter of 1998-99. They are the so called high net worth players — corporates and individuals — who in Khatau's terms, 'are aggressive about managing their wealth, and look closely at comparative performance'. While their fastidiousness has forced them to pick the private sector mutual funds, whose disclosures and performance has both been ahead of their public sector cousins, their interest does not represent every investor's disillusionment.


Q. The amount of money entrusted to the care of the mutual funds was


  1. Rs. 75,000 crore.

  2. Rs. 80,000 crore

  3. Rs. 85,000 crore.

  4. Rs. 82,000 crore.


Ans . C


Q. The end of mutual funds was carried out at the hands of


  1. the government.

  2. non-banking finance companies.

  3. the individual investors

  4. banks.


Ans . C


Q. According to the passage, the flaws of the mutual funds lay in their


  1. post-liberalisation syndrome.

  2. imprudent and irresponsible handling.

  3. stagnation.

  4. all of these


Ans . B


Q. According to the passage, one of the reasons for the failure of the mutual funds was


  1. their indisciplined approach to investment

  2. their devil-may-care approach to the world of finance.

  3. their ability to deceive investors.

  4. their inability to read the pulse of their investors.


Ans . A


Q. According to the writer, one of the fallouts of the end of mutual funds is that


  1. at least some of the big schemes due for redemption over the next three years will be unable to service their investors.

  2. only very few of the big schemes due for redemption over the next three years will be unable to service their investors.

  3. none of the big schemes due for redemption over the next three years will be able to service their investors.

  4. None of these


Ans . A


Q. It can be inferred from the passage that


  1. money was siphoned away outside the country by the mutual funds.

  2. many of the mutual fund offices indulged in malpractice.

  3. money invested in the mutual fund schemes were never returned to the investors.

  4. a sustained attack by the media exposed the anomalies in the mutual fund industry.


Ans . B


Q. The current rank of the mutual fund industry in terms of safety and returns on deposits respectively is


  1. third and fourth.

  2. tenth and twelfth.

  3. fourth and fifth.

  4. it is not ranked at all


Ans . C


Q. The increase in the number of cell phone subscriptions in the office of an asset management company was due to the fact that


  1. calls made by employees for personal deals couldn't be lodged in the company's records.

  2. employees found it easier to deal with investors without involving the company.

  3. the company was scrupulous about maintaining correct records.

  4. the company was unscrupulous in granting personal deals to the employees.


Ans . A


Q. According to the passage, mutual funds caused a loss of


  1. Rs. 10,000 crore of the investors' money.

  2. Rs. 11,000 crore of the investors' money.

  3. Rs. 5,000 crore of the investors' money.

  4. Rs. 8,000 crore of the investors' money.


Ans . B


Q. On the basis of the passage, it may be said that, in terms of retrieving their money, the investors


  1. are caught between the devil and the deep blue sea.

  2. have a no-exit route.

  3. have to make do with little or no gain.

  4. will trust the few bright stars in the mutual fund industry.


Ans . A


Q. According to the passage, one of the reasons for the euphoria in the mutual fund industry can be attributed to


  1. the stock market boom in the late eighties and early nineties.

  2. failure of the primary market.

  3. Both (a) and (b)

  4. Neither (a) nor (b)


Ans . A


Reading comprehensions


The invention of the gas turbine by Frank Whittle in England and Hans von Ohain in Germany in 1939 signalled the beginning of jet transport. Although the French engineer Lorin had visualized the concept of jet propulsion more than 25 years earlier, it took improved materials and the genius of Whittle and von Ohain to recognize the advantage that a gas turbine offered over a piston engine, including speeds in excess of 350 miles per hour. The progress from the first flights of liquid propellant rocket and jet-propelled aircraft in 1939 to the first faster-than-sound (supersonic) manned airplane (the Bell X-1) in 1947 happened in less than a decade. This then led very rapidly to a series of supersonic fighters and bombers, the first of which became operational in the 1950s.

World War II technology foundations and emerging Cold War imperatives then led us into space with the launch of Sputnik in 1957 and the placing of the first man on the moon only 12 years later — a mere 24 years after the end of World War II. Now a hypersonic flight can take you anywhere in the planet in less than four hours. British Royal Air Force and Royal Navy and the air forces of several other countries are going to use a single-engine cousin to the F/A-22, called the F-35 Joint Strike Fighter. These planes exhibit stealthy angles and coatings that make it difficult for radar to detect them, among aviation's most cutting-edge advances in design. The V-22, known as tilt-rotor, part helicopter, part airplane, takes off vertically, then tilts its engine forward for winged flight.

It provides speed, three times the payload, five times the range of the helicopters it's meant to replace. The new fighter, F/A-22 Raptor, with more than a million parts, shows a perfect amalgamation of stealth, speed, avionics and agility. It seems conventional forms, like the Predator and Global Hawk are passé, the stealthier unmanned aerial vehicles (UAVs) are in. They are shaped like kites, bats and boomerang, all but invisible to the enemy radar and able to remain over hostile territory without any fear of getting grilled if shot down.

Will the UAVs take away pilots' jobs permanently? Can a computer-operated machine take a smarter and faster decision in a war-like situation? The new free-flight concept will probably supplement the existing air traffic control system by computers on each plane to map the altitude, route, weather and other planes; and a decade from now, there will be no use of radar any more. How much bigger can the airplanes get? In the '50s they got speed, in the '80s they became stealthy. Now they are getting smarter thanks to computer automation. The change is quite huge: from the four-seater to the A380 airplane.

It seems we are now trading speed for size as we build a new superjumbo jet, the 555 seater A380, which will fly at almost the same speed of the Boeing 707, introduced half a century ago, but with an improved capacity, range, greater fuel economy. A few years down the line will come the truly larger model, to be known as 747X. In the beginning of 2005, the A380, the world's first fully double-decked superjumbo passenger jet, weighing 1.2 million pounds, may carry a load of about 840 passengers.

Barring the early phase, civil aviation has always lagged behind the military technologies (of jet engines, lightweight composite materials, etc.). There are two fundamental factors behind the decline in commercial aeronautics in comparison to military aeronautics. There is no collective vision of our future such as the one that drove us in the past. There is also a need for a more aggressive pool of airplane design talents to maintain an industry that continues to find a multibillion dollar-a-year market for its product.

Can the history of aviation technology tell us something about the future of aeronautics? Have we reached a final state in our evolution to a mature technology in aeronautics? Are the challenges of coming out with the 'better, cheaper, faster' designs somehow inferior to those that are suited for 'faster, higher, further'? Safety should improve greatly as a result of the forthcoming improvements in airframes, engines, and avionics. Sixty years from now, aircraft will recover on their own if the pilot loses control.

Satelites are the key not only to GPS (global positioning system) navigation but also to in-flight communications, uplinked weather, and even in-flight e-mail. Although there is some debate about what type of engines will power future airplanes — lightweight turbines, turbocharged diesels, or both — there is little debate about how these power plants will be controlled. Pilots of the future can look forward to more and better on-board safety equipment.


Q. Why might radars not be used a decade from now?


  1. Stealth technology will advance so much that it is pointless to use radar to detect aircraft.

  2. UAVs can remain over hostile territory without any danger of being detected.

  3. Computers on board may enable aircraft to manage safe navigation on their own

  4. It is not feasible to increase the range of radars.


Ans . C


Q. According to the author, commercial aeronautics, in contrast to military aeronautics, has declined because, among other things.


  1. Speed and technology barriers are more easily overcome in military aeronautics.

  2. The collective vision of the past continues to drive civil and commercial aeronautics

  3. Though the industry has a huge market, it has not attracted the right kind of aircraft designers.

  4. There is a shortage of materials, like light weight composites, used in commercial aeronautics.


Ans . C


Q. According to the first paragraph of the passage, which of the following statements is NOT false?


  1. Frank Whittle and Hans von Ohain were the first to conceive of jet propulsion.

  2. Supersonic fighter planes were first used in World War II.

  3. No man had travelled faster than sound until the 1950s.

  4. The exploitation of jet propulsion for supersonic aviation has been remarkably fast.


Ans . D


Q. What is the fourth paragraph of the passage, starting, “How much bigger . . . ”, about?


  1. Stealth, speed, avionics, and agility of new aircraft.

  2. The way aircraft size has been growing.

  3. Use of computer automation in aircraft.

  4. Super-jumbo jets that can take more than 500 passengers.


Ans . B


Q. What is the most noteworthy difference between V-22 and a standard airplane?


  1. It can take off vertically.

  2. It has winged flight.

  3. It has excellent payload.

  4. Its range is very high.


Ans . A


Reading comprehension


Pure love of learning, of course, was a less compelling motive for those who became educated for careers other than teaching. Students of law in particular had a reputation for being materialistic careerists in an age when law was becoming known as 'the lucrative science' and its successful practice the best means for rapid advancement in the government of both church and state. Medicine too had its profit-making attractions.

Those who did not go on to law or medicine could, if they had been well trained in the arts, gain positions at royal courts or rise in the clergy. Eloquent testimony to the profit motive behind much of 12thcentury education was the lament of a student of Abelard around 1150: “Christians educate their sons . . . for gain, in order that the one brother, if he be a clerk, may help his father and mother and his other brothers, saying that a clerk will have no heir and whatever he has will be ours and the other brothers.”

With the opening of positions in law, government and the church, education became a means for advancement not only in income but also in status. Most who were educated were wealthy, but in the 12th century, more often than before, many were not and were able to rise through the ranks by means of their education.

The most familiar examples are Thomas Becket, who rose from a humble background to become chancellor of England and then archbishop of Canterbury, and John of Salisbury, who was born a 'plebeian' but because of his reputation for learning died as bishop of Chartres. The instances of Becket and John of Salisbury bring us to the most difficult question concerning 12thcentury education:

To what degree was it still a clerical preserve? Despite the fact that throughout the 12th century the clergy had a monopoly of instruction, one of the outstanding medievalists of our day, R. W. Southern, refers with good reason to the institutions staffed by the clergy as 'secular schools'. How can we make sense out of the paradox that 12th-century schools were clerical and yet 'secular'? Let us look at the clerical side first. Not only were all 12th-century teachers except professionals and craftsmen in church order, but in northern Europe students in schools had clerical status and looked like priests. Not that all really were priests, but by virtue of being students all were awarded the legal privileges accorded to the clergy.

Furthermore, the large majority of 12th-century students, outside of the possible exception of Italy, if not already priests became so after their studies were finished. For these reasons, the term 'cleric' was often used to denote a man who was literate and the term 'layman' one who was illiterate. The English word for cleric, clerk, continued for a long time to be a synonym for student or for a man who could write, while the French word clerc even today has the connotation of intellectual.

Despite all this, 12th-century education was taking on many secular qualities in its environment, goals, and curriculum. Student life obviously became more secular when it moved out from the monasteries into the bustling towns. Most students wandered from town to town in search not only of good masters but also of worldly excitement, and as the 12th century progressed they found the best of each in Paris. More important than environment was the fact that most students, even though they entered the clergy, had secular goals. Theology was recognized as the 'queen of the sciences', but very few went on to it.

Instead they used their study of the liberal arts as a preparation for law, medicine, government service, or advancement in the ecclesiastical hierarchy. This being so, the curriculum of the liberal arts became more sophisticated and more divorced from religion. Teaching was still almost exclusively in Latin, and the first book most often read was the Psalter, but further education was no longer similar to that of a choir school. In particular, the discipline of rhetoric was transformed from a linguistic study into instruction in how to compose letters and documents;

there was a new stress on logic; and in all the liberal arts and philosophy texts more advanced than those known in the early Middle Ages were introduced. Along with the rise of logic came the translation of Greek and Arabic philosophical and scientific works. Most important was the translation of almost all the writings of Aristotle, as well as his sophisticated Arabic commentators, which helped to bring about an intellectual revolution based on Greek rationalism.

On a more prosaic level, contact with Arabs resulted in the introduction in the 12th century of the Arabic numeral system and the concept of zero. Though most westerners first resisted this and made crude jokes about the zero as an ambitious number 'that counts for nothing and yet wants to be counted', the system steadily made its inroads first in Italy and then throughout Europe, thereby vastly simplifying the arts of computation and record-keeping.


Q. According to the passage, what led to the secularisation of the curriculum of the liberal arts in the 12th century?


  1. It was divorced from religion and its influences.

  2. Students used it mainly as a base for studying law and medicine.

  3. Teaching could no longer be conducted exclusively in Latin.

  4. Arabic was introduced into the curriculum


Ans . A


Q. According to the author, in the 12th century, individuals were motivated to get higher education because it


  1. was a means for material advancement and higher status.

  2. gave people with wealth an opportunity to learn.

  3. offered a coveted place for those with a love of learning

  4. directly added to the income levels of people.


Ans . A


Q. According to the passage, 12th-century schools were clerical and yet secular because


  1. many teacher were craftsmen and professionals who did not form part of the church.

  2. while the students had the legal privileges accorded to the clergy and looked like priests, not all were really priests.

  3. the term 'cleric' denoted a literate individual rather than a strict association with the church.

  4. though the clergy had a monopoly in education, the environment, objectives and curriculum in the schools were becoming secular.


Ans . D


Q. What does the sentence 'Christians educate their sons . . . will be ours and the other brothers' imply?


  1. The Christian family was a close-knit unit in the 12th century.

  2. Christians educated their sons not so much for the love of learning as for material gain

  3. Christians believed very strongly in educating their sons in the Church

  4. The relationship between Christian parents and their sons was exploitative in the 12th century.


Ans . B


Q. According to the passage, which of the following is the most noteworthy trend in education in 12thcentury Europe?


  1. Secularization of education.

  2. Flowering of theology as the queen of the sciences.

  3. Wealthy people increasingly turning to education.

  4. Rise of the clergy's influence on the curriculum.


Ans . A


Reading comprehension


At first sight, it looks as though panchayati raj, the lower layer of federalism in our polity, is as firmly entrenched in our system as is the older and higher layer comprising the Union Government and the State. Like the democratic institutions at the higher level, those at the panchayat level, the panchayati raj institutions (PRIs), are written into and protected by the Constitution. All the essential features, which distinguish a unitary system from a federal one, are as much enshrined at the lower as at the upper level of our federal system. But look closely and you will discover a fatal flaw. The letter of the Constitution as well as the spirit of the present polity have exposed the intra-State level of our federal system to a dilemma of which the inter-State and Union-State layers are free. The flaw has many causes. But all of them are rooted in an historical anomaly, that while the dynamics of federalism and democracy have given added strength to the rights given to the States in the Constitution, they have worked against the rights of panchayats. At both levels of our federal system there is the same tussle between those who have certain rights and those who try to encroach upon them if they believe they can. Thus, the Union Government was able to encroach upon certain rights given to the States by the Constitution. It got away with that because the single dominant party system, which characterised Centre-State relations for close upon two decades, gave the party in power at the Union level many extra-constitutional political levers. Second, the Supreme Court had not yet begun to extend the limits of its power. But all that has changed in recent times. The spurt given to a multi-party democracy by the overthrow of the Emergency in 1977 became a long-term trend later on because of the ways in which a vigorously democratic multi-party system works in a political society which is as assertively pluralistic as Indian society is. It gives political clout to all the various segments which constitute that society. Secondly, because of the linguistic reorganisation of States in the 1950s, many of the most assertive segments have found their most assertive expression as States. Thirdly, with single-party dominance becoming a thing of the past at the Union level, governments can be formed at that level only by multi-party coalitions in which State-level parties are major players. This has made it impossible for the Union Government to do much about anything unless it also carries a sufficient number of State-level parties with it. Indian federalism is now more real than it used to be, but an unfortunate side-effect is that India's panchayati raj system, inaugurated with such fanfare in the early 1980s, has become less real. By the time the PRIs came on the scene, most of the political space in our federal system had been occupied by the Centre in the first 30 years of Independence, and most of what was still left after that was occupied by the States in the next 20. PRIs might have hoped to wrest some space from their immediate neighbour, the States, just as the States had wrested some from the Centre. But having at last managed to checkmate the Centre's encroachments on their rights, the States were not about to allow the PRIs to do some encroaching of their own. By the 1980's and early 1990s, the only nationally left, the Congress, had gone deeper into a siege mentality. Finding itself surrounded by State-level parties, it had built walls against them in stead of winning them over. Next, the States retaliated by blocking Congress proposals for panchayati raj in Parliament, suspecting that the Centre would try to use panchayats to by-pass State Governments. The suspicion fed on the fact that the powers proposed by the Congress for panchayats were very similar to many of the more lucrative powers of State Governments. State-level leaders also feared, perhaps, that if panchayat-level leaders captured some of the larger PRIs, such as district-level panchayats, they would exert pressure on State-level leaders through intra-State multi-party federalism. It soon became obvious to Congress leaders that there was no way the panchayati raj amendments they wanted to write into the Constitution would pass muster unless State-level parties were given their pound of flesh. The amendments were allowed only after it was agreed that the powers of panchayats could be listed in the Constitution. Illustratively, they would be defined and endowed on PRIs by the State Legislature acting at its discretion. This left the door wide open for the States to exert the power of the new political fact that while the Union and State Governments could afford to ignore panchayats as long as the MLAs were happy, the Union Government had to be sensitive to the demands of State-level parties. This has given State-level actors strong beachheads on the shores of both inter-State and intra-State federalism. By using various administrative devices and non-elected parallel structures, State Governments have subordinated their PRIs to the State administration and given the upper hand to State Government officials against the elected heads of PRIs. Panchayats have become local agencies for implementing schemes drawn up in distant State capitals. And their own volition has been further circumscribed by a plethora of 'Centrally-sponsored schemes'. These are drawn up by even more distant Central authorities but at the same time tie up local staff and resources on pain of the schemes being switched off in the absence of matching local contribution. The 'foreign aid' syndrome can be clearly seen at work behind this kind of 'grass roots development'.


Q. The central theme of the passage can be best summarized as


  1. our grassroots development at the panchayat level is now driven by the 'foreign aid' syndrome.

  2. panchayati raj is firmly entrenched at the lower level of our federal system of governance.

  3. a truly federal polity has not developed since PRIs have not been allowed the necessary political space.

  4. the Union Government and State-level parties are engaged in a struggle for the protection of their respective.


Ans . C


Q. The sentence in the last paragraph, “And their own volition has been further circumscribed. . .” refers to


  1. the weakening of the local institutions' ability to plan according to their needs.

  2. the increasing demands made on elected local leaders to match central grants with local contributions.

  3. the empowering of the panchayat system as implementers of schemes from State capitals.

  4. the process by which the prescribed Central schemes are reformulated by local elected leaders.


Ans . A


Q. What is the 'dilemma' at the intra-State level mentioned in the first paragraph of the passage?


  1. Should the state governments wrest more space from the Union, before considering the panchayati system?

  2. Should the rights similar to those that the States managed to get be extended to panchayats as well?

  3. Should the single party system which has withered away be brought back at the level of the States?

  4. Should the States get 'their pound of flesh' before allowing the Union Government to pass any more laws?


Ans . B


Q. Which of the following most closely describes the 'fatal flaw' that the passage refers to?


  1. The ways in which the democratic multi-party system works in an assertively pluralistic society like India's are flawed.

  2. The mechanisms that our federal system uses at the Union Government level to deal with States are imperfect.

  3. The instruments that have ensured federalism at one level, have been used to achieve the opposite at another

  4. The Indian Constitution and the spirit of the Indian polity are fatally flawed


Ans . C


Q. Which of the following best captures the current state of Indian federalism as described in the passage?


  1. The Supreme Court has not begun to extend the limits of its power.

  2. The multi-party system has replaced the single party system.

  3. The Union, State and panchayati raj levels have become real.

  4. There is real distribution of power between the Union and State-level parties.


Ans . B


Reading comprehensions


While I was in class at Columbia, struggling with the esoterica du jour, my father was on a bricklayer's scaffold not far up the street, working on a campus building. Once we met up on the subway going home — he was with his tools, I with my books. My father wasn't interested in Thucydides, and I wasn't up on arches. My dad has built lots of places in New York City he can't get into: colleges, condos, office towers.

He made his living on the outside. Once the walls were up, a place took on a different feel for him, as though he wasn't welcome anymore. Related by blood, we're separated by class, my father and I. Being the white-collar child of a blue-collar parent means being the hinge on the door between two ways of life. With one foot in the working-class, the other in the middle class, people like me are Straddlers, at home in neither world, living a limbo life.

What drove me to leave what I knew? Born blue-collar, I still never felt completely at home among the tough guys and anti-intellectual crowd of my neighbourhood in deepest Brooklyn. I never did completely fit in among the preppies and suburban royalty of Columbia, either. It's like that for Straddlers. It was not so smooth jumping from Italian old-world style to US professional in a single generation. Others who were the first in their families to go to college, will tell you the same thing: the academy can render you unrecognisable to the very people who launched you into the world.

The ideas and values absorbed in college challenge the mom-and pop orthodoxy that passed for truth for 18 years. Limbo folk may eschew polyester blends for sea-isle cotton, prefer Brie to Kraft slices. They marry outside the neighbourhood and raise their kids differently. They might not be in church on Sunday. When they pick careers (not jobs), it's often a kind of work their parents never heard of or can't understand. But for the white-collar kids of blue-collar parents, the office is not necessarily a sanctuary.

In Corporate America, where the rules are based on notions foreign to working-class people, a Straddler can get lost. Social class counts at the office, even though nobody likes to admit it. Ultimately, corporate norms are based on middle-class values, business types say. From an early age, middle-class people learn how to get along, using diplomacy, nuance, and politics to grab what they need. It is as though they are following a set of rules laid out in a manual that blue-collar families never have the chance to read.

People born into the middle class to parents with college degrees have lived lives filled with what French sociologist Pierre Bourdieu calls 'cultural capital'. Growing up in an educated environment, they learn about Picasso and Mozart, stock portfolios and crème brulee. In a home with cultural capital, there are networks: someone always has an aunt or golfing buddy with the inside track for an internship or some entry-level job.

Dinner-table talk could involve what happened that day to mom and dad at the law firm, the doctor's office, or the executive suite. Middle-class kids can grow up with a sense of entitlement that will carry them through their lives. This 'belongingness' is not just related to having material means, it also has to do with learning and possessing confidence in your place in the world. Such early access and direct exposure to culture in the home is the more organic, 'legitimate' means of appropriating cultural capital, Bourdieu tells us.

Those of us possessing 'ill-gotten Culture' can learn it, but never as well. Something is always a little off about us, like an engine with imprecise timing. There's a greater match between middleclass lives and the institutions in which the middle class works and operates — universities or corporations. Children of the middle and upper classes have been speaking the language of the bosses and supervisors forever.

Blue-collar kids are taught by their parents and communities to work hard to achieve, and that merit is rewarded. But no blue-collar parent knows whether such things are true in the middle-class world. Many professionals born to the working-class report feeling out of place and out of place and outmanoeuvred in the office. Soon enough, Straddlers learn that straight talk won't always cut. Resolving conflicts head-on and speaking your mind doesn't always work, no matter how educated the Straddler is.

In the working-class, people perform jobs in which they are closely supervised and are required to follow orders and instructions. That, in turn, affects how they socialise their children. Children of the workingclass are brought up in a home in which conformity, obedience and intolerance for back talk are the norm — the same characteristics that make a good factory worker.


Q. According to the passage, which of the following statements about 'cultural capital' is NOT true?


  1. It socializes children early into the norms of middle class institutions.

  2. It helps them learn the language of universities and corporations.

  3. It creates a sense of enlightenment in middle-class children.

  4. it develops bright kids into Straddlers


Ans . D


Q. According to the passage, the patterns of socialization of working-class children make them most suited for jobs that require


  1. diplomacy

  2. compliance with orders.

  3. enterprise and initiative.

  4. high risk-taking.


Ans . B


Q. When Straddlers enter white collar jobs, they get lost because


  1. they are thrown into an alien value system

  2. their families have not read the rules in corporate manuals.

  3. they have no one to guide them through the corporate maze.

  4. they miss the 'mom and pop orthodoxy'.


Ans . A


Q. What does the author's statement, “My father wasn't interested in Thucydides, and I wasn't up on arches,” illustrate?


  1. Organic cultural capital

  2. Professional arrogance and social distance

  3. Evolving social transformation

  4. Breakdown of family relationships


Ans . C


Q. Which of the following statements about Straddlers does the passage NOT support explicitly?


  1. Their food preferences may not match those of their parents

  2. They may not keep up some central religious practices of their parents.

  3. They are at home neither in the middle class nor in the working-class.

  4. Their political ideologies may differ from those of their parents.


Ans . D


Reading comprehensions


The endless struggle between the flesh and the spirit found an end in Greek art. The Greek artists were unaware of it. They were spiritual materialists, never denying the importance of the body and ever seeing in the body a spiritual significance. Mysticism on the whole was alien to the Greeks, thinkers as they were. Thought and mysticism never go well together and there is little symbolism in Greek art.

Athena was not a symbol of wisdom but an embodiment of it and her statues were beautiful grave women, whose seriousness might mark them as wise, but who were marked in no other way. The Apollo Belvedere is not a symbol of the sun, nor the Versailles Artemis of the moon. There could be nothing less akin to the ways of symbolism than their beautiful, normal humanity. Nor did decoration really interest the Greeks. In all their art they were preoccupied with what they wanted to express, not with ways of expressing it, and lovely expression, merely as lovely expression, did not appeal to them at all. Greek art is intellectual art, the art of men who were clear and lucid thinkers, and it is therefore plain art.

Artists than whom the world has never seen greater, men endowed with the spirit's best gift, found their natural method of expression in the simplicity and clarity which are the endowment of the unclouded reason. “Nothing in excess,” the Greek axiom of art, is the dictum of men who would brush aside all obscuring, entangling superfluity, and see clearly, plainly, unadorned, what they wished to express.

Structure belongs in an especial degree to the province of the mind in art, and architectonics were pre-eminently a mark of the Greek. The power that made a unified whole of the trilogy of a Greek tragedy, that envisioned the sure, precise, decisive scheme of the Greek statue, found its most conspicuous expression in Greek architecture. The Greek temple is the creation, par excellence, of mind and spirit in equilibrium.

A Hindoo temple is a conglomeration of adornment. The lines of the building are completely hidden by the decorations. Sculptured figures and ornaments crowd its surface, stand out from it in thick masses, break it up into a bewildering series of irregular tiers. It is not a unity but a collection, rich, confused. It looks like something not planned but built this way and that as the ornament required. The conviction underlying it can be perceived: each bit of the exquisitely wrought detail had a mystical meaning and the temple's exterior was important only as a means for the artist to inscribe thereon the symbols of the truth. It is decoration, not architecture.

Again, the gigantic temples of Egypt, those massive immensities of granite which look as if only the power that moves in the earthquake were mighty enough to bring them into existence, are something other than the creation of geometry balanced by beauty. The science and the spirit are there, but what is there most of all is force, unhuman force, calm but tremendous, overwhelming. It reduces to nothingness all that belongs to man. He is annihilated. The Egyptian architects were possessed by the consciousness of the awful, irresistible domination of the ways of nature; they had no thought to give to the insignificant atom that was man.

Greek architecture of the great age is the expression of men who were, first of all, intellectual artists, kept firmly within the visible world by their mind, but, only second to that, lovers of the human world. The Greek temple is the perfect expression of the pure intellect illumined by the spirit. No other great buildings anywhere approach its simplicity. In the Parthenon straight columns rise to plain capitals; a pediment is sculptured in bold relief;

there is nothing more. And yet — here is the Greek miracle — this absolute simplicity of structure is alone in majesty of beauty among all the temples and cathedrals and palaces of the world. Majestic but human, truly Greek. No superhuman force as in Egypt; no strange supernatural shapes as in India; the Parthenon is the home of humanity at ease, calm, ordered, sure of itself and the world. The Greeks flung a challenge to nature in the fullness of their joyous strength. They set their temples on the summit of a hill overlooking the wide sea, outlined against the circle of the sky.

They would build what was more beautiful than hill and sea and sky and greater than all these. It matters not at all if the temple is large or small; one never thinks of the size. It matters not how much it is in ruins. A few white columns dominate the lofty height at Sunion as securely as the great mass of the Parthenon dominates all the sweep of sea and land around Athens. To the Greek architect man was the master of the world. His mind could understand its laws; his spirit could discover its beauty.


Q. “The Greeks flung a challenge to nature in the fullness of their joyous strength.” Which of the following best captures the 'challenge' that is being referred to?


  1. To build a monument matching the background colours of the sky and the sea.

  2. To build a monument bigger than nature's creations.

  3. To build monuments that were more appealing to the mind and spirit than nature's creations.

  4. To build a small but architecturally perfect monument.


Ans . C


Q. Which of the following is NOT a characteristic of Greek architecture, according to the passage


  1. A lack of excess

  2. Simplicity of form

  3. Expression of intellect

  4. Mystic spirituality


Ans . D


Q. From the passage, which of the following combinations can be inferred to be correct?


  1. Hindoo temple — power of nature

  2. Parthenon — simplicity

  3. Egyptian temple — mysticism

  4. Greek temple — symbolism


Ans . B


Q. According to the passage, what conception of man can be inferred from Egyptian architecture?


  1. Man is the centre of creation.

  2. Egyptian temples save man from unhuman forces

  3. Temples celebrate man's victory over nature.

  4. Man is inconsequential before the tremendous force of nature.


Ans . D


Q. According to the passage, which of the following best explains why there is little symbolism in Greek art?


  1. The Greeks focused on thought rather than mysticism.

  2. The struggle between the flesh and the spirit found an end in Greek art.

  3. Greek artists were spiritual materialists.

  4. Greek statues were embodiments rather than symbols of qualities


Ans . A


A game of strategy, as currently conceived in game theory, is a situation in which two or more “players” make choices among available alternatives (moves). The totality of choices determines the outcomes of the game, and it is assumed that the rank order of preferences for the outcomes is different for different players.

Thus the “interests” of the players are generally in conflict. Whether these interests are diametrically opposed or only partially opposed depends on the type of game. Psychologically, most interesting situations arise when the interests of the players are partly coincident and partly opposed, because then one can postulate not only a conflict among the players but also inner conflicts within the players. Each is torn between a tendency to cooperate, so as to promote the common interests, and a tendency to compete, so as to enhance his own individual interests. Internal conflicts are always psychologically interesting. What we vaguely call “interesting” psychology is in very great measure the psychology of inner conflict.

Inner conflict is also held to be an important component of serious literature as distinguished from less serious genres. The classical tragedy, as well as the serious novel reveals the inner conflict of central figures. The superficial adventure story on the other hand, depicts only external conflict; that is, the threats to the person with whom the reader (or viewer) identifies stem in these stories exclusively from external obstacles and from the adversaries who create them. On the most primitive level this sort of external conflict is psychologically empty. In the fisticuffs between the protagonists of good and evil, no psychological problems are involved or, at any rate, none are depicted in juvenile representations of conflict. The detective story, the “adult” analogue of a juvenile adventure tale, has at times been described as a glorification of intellectualized conflict.

However, a great deal of the interest in the plots of these stories is sustained by withholding the unraveling of a solution to a problem. The effort of solving the problem is in itself not a conflict if the adversary (the unknown criminal) remains passive, like Nature, whose secrets the scientist supposedly unravels by deduction. If the adversary actively puts obstacles in the detective’s path toward the solution, there is genuine conflict. But the conflict is psychologically interesting only to the extent that it contains irrational components such as a tactical error on the criminal’s part or the detective’s insight into some psychological quirk of the criminal or something of this sort. Conflict conducted in a perfectly rational manner is psychologically no more interesting than a standard Western.

For example, Tic-tac-toe, played perfectly by both players, is completely devoid of psychological interest. Chess may be psychologically interesting but only to the extent that it is played not quite rationally. Played completely rationally, chess would not be different from Tic-tac-toe. In short, a pure conflict of interest (what is called a zero-sum game) although it offers a wealth of interesting conceptual problems, is not interesting psychologically, except to the extent that its conduct departs from rational norms.



Q. According to the passage, internal conflicts are psychologically more interesting than external conflicts because


  1. internal conflicts, rather than external conflicts, form an important component of serious literature as distinguished from less serious genres

  2. only juveniles or very few “adults” actually experience external conflict, while internal conflict is more widely prevalent in society.

  3. in situations of internal conflict, individuals experience a dilemma in resolving their own preferences for different outcomes.

  4. there are no threats to the reader (or viewer) in case of external conflicts.


Ans . C



Q. Which, according to the author, would qualify as interesting psychology?


  1. A statistician’s dilemma over choosing the best method to solve an optimization problem.

  2. A chess player’s predicament over adopting a defensive strategy against an aggressive opponent.

  3. A mountaineer’s choice of the best path to Mt. Everest from the base camp.

  4. A finance manager’s quandary over the best way of raising money from the market.


Ans . B



Q. According to the passage, which of the following options about the application of game theory to a conflict-of-interest situation is true?


  1. Assuming that the rank order of preferences for options is different for different players.

  2. Accepting that the interests of different players are often in conflict.

  3. Not assuming that the interests are in complete disagreement

  4. All of the above.


Ans . B



Q. The problem solving process of a scientist is different from that of a detective because


  1. scientists study inanimate objects, while detectives deal with living criminals or law offenders.

  2. scientists study known objects, while detectives have to deal with unknown criminals or law offenders.

  3. scientists study phenomena that are not actively altered, while detectives deal with phenomena that have been deliberately influenced to mislead

  4. scientists study psychologically interesting phenomena, while detectives deal with “adult” analogues of juvenile adventure tales


Ans . C





Crinoline and croquet are out. As yet, no political activists have thrown themselves in front of the royal horse on Derby Day. Even so, some historians can spot the parallels. It is a time of rapid technological change. It is a period when the dominance of the world’s superpower is coming under threat. It is an epoch when prosperity masks underlying economic strain. And, crucially, it is a time when policy-makers are confident that all is for the best in the best of all possible worlds. Welcome to the Edwardian Summer of the second age of globalisation. Spare a moment to take stock of what’s been happening in the past few months. Let’s start with the oil price, which has rocketed to more than $65 a barrel, more than double its level 18 months ago.

The accepted wisdom is that we shouldn’t worry our little heads about that, because the incentives are there for business to build new production and refining capacity, which will effortlessly bring demand and supply back into balance and bring crude prices back to $25 a barrel. As Tommy Copper used to say, ‘just like that’. Then there is the result of the French referendum on the European Constitution, seen as thick-headed luddites railing vainly against the modern world. What the French needed to realize, the argument went, was that there was no alternative to the reforms that would make the country more flexible, more competitive, more dynamic. Just the sort of reforms that allowed Gate Gourmet to sack hundreds of its staff at Heathrow after the sort of ultimatum that used to be handed out by Victorian mill owners.

An alternative way of looking at the French “non” is that our neighbours translate “flexibility” as “you’re fired”. Finally, take a squint at the United States. Just like Britian a century ago, a period of unquestioned superiority is drawing to a close. China is still a long way from matching America’s wealth, but it is growing at a stupendous rate and economic strength brings geo-political clout. Already, there is evidence of a new scramble for Africa as Washington and Beijing compete for oil stocks. Moreover, beneath the surface of the US economy, all is not well.

Growth looks healthy enough, but the competition from China and elsewhere has meant the world’s biggest economy now imports far more than it exports. The US is living beyond its means, but in this time of studied complacency a current account deficit worth 6 perfect of gross domestic product is seen as a sign of strength, not weakness. In this new Edwardian summer, comfort is taken from the fact that dearer oil has not had the savage inflationary consequences of 1973-1974, when a fourfold increase in the cost of crude brought an abrupt end to a postwar boom that had gone on uninterrupted for a quarter of a century. True, the cost of living has been affected by higher transport costs, but we are talking of inflation at 2.3 per cent and not 27 per cent.

Yet the idea that higher oil prices are of little consequence is fanciful. If people are paying more to fill up their cars it leaves them with less to spend on everything else, but there is a reluctance to consume less. In the 1970s unions were strong and able to negotiate large, compensatory pay deals that served to intensify inflationary pressure. In 2005, that avenue is pretty much closed off, but the abolition of all the controls on credit that existed in the 1970s means that households are invited to borrow more rather than consume less. The knock-on effects of higher oil prices are thus felt in different ways – through high levels of indebtedness, in inflated asset prices, and in balance of payments deficits.

There are those who point out, rightly, that modern industrial capitalism has proved mightily resilient these past 250 years, and that a sign of the enduring strength of the system has been the way it apparently shrugged off everything – a stock market crash, 9/11, rising oil prices – that have been thrown at it in the half decade since the millennium. Even so, there are at least three reasons for concern. First, we have been here before. In terms of political economy, the first era of globalisation mirrored our own.

There was a belief in unfettered capital flows, in free migration. Eventually, though, there was a backlash, manifested in a struggle between free traders and protectionists, and in rising labour militancy. Second, the world is traditionally as its most fragile at times when the global balance of power is in flux. By the end of the nineteenth century, Britain’s role as the hegemonic power was being challenged by the rise of the United States, Germany, and Japan while the Ottoman and Hapsburg empires were clearly in rapid decline.

Looking ahead from 2005, it is clear that over the next two or three decades, both China and India – which together account for half the world’s population – will flex their muscles. Finally, there is the question of what rising oil prices tell us. The emergence of China and India means global demand for crude is likely to remain high a t a time when experts say production is about to top out. If supply constraints start to bite, any decline in the prices are likely to be short-term cyclical affairs punctuating a long upward trend.


Q. By the expression ‘Edwardian Summer’, the author refers to a period in which there is


  1. unparalleled luxury and opulence

  2. a sense of complacency among people because of all-round prosperity

  3. a culmination of all-round economic prosperity.

  4. an imminent danger lurking behind economic prosperity.


Ans . B



Q. What, according to the author, has resulted in a widespread belief in the resilence of modern capitalism?


  1. Growth in the economies of Western countries despite shocks in the form of increase in levels of indebtedness and inflated asset prices

  2. increase in the prosperity of Western countries and China despite rising oil prices

  3. Continued growth of Western economies despite a rise in terrorism, an increase in oil prices and other similar shocks.

  4. The success of continued reforms aimed at making Western economies more dynamic, competitive and efficient.


Ans . C



Q. Which of the following best represents the key argument made by the author?


  1. The rise in oil prices, the flux in the global balance of power and historical precedents should make us question our belief that the global economic prosperity would continue

  2. The belief that modern industrial capitalism is highly resilient and capable of overcoming shocks will be belied soon

  3. Widespread prosperity leads to neglect of early signs of underlying economic weakness, manifested in higher oil prices and a flux in the global balance of power.

  4. A crisis is imminent in the West given the growth of countries like China and India and the increase in oil prices.


Ans . A



Q. What can be inferred about the author’s view when he states ‘As Tommy Cooper used to say “just like that”?


  1. Industry has incentives to build new production and refining capacity and therefore oil prices would reduce.

  2. There would be a correction in the price levels of oil once new production capacity is added.

  3. The decline in oil prices is likely to be short-term in nature.

  4. It is not necessary that oil prices would go down to earlier levels.


Ans . D



While complex in the extreme, Derrida’s work has proven to be a particularly influential approach to the analysis of the ways in which language structures our understanding of ourselves and the world we inhabit, an approach he termed deconstruction. In its simplest formulation, deconstruction can be taken to refer to a methodological strategy which seeks to uncover layers of hidden meaning in a text that have been denied or suppressed.

The term ‘text’, in this respect, does not refer simply to a written form of communication, however. Rather, texts are something we all produce and reproduce constantly in our every day social relations, be they spoken, written or embedded in the construction of material artifacts. At the heart of Derrida’s deconstructive approach is his critique of what he perceives to be the totalitarian impulse of the Enlightenment pursuit to bring all that exists in the world under the domain of representative language, a pursuit he refers to as logocentrism.

Logocentrism is the search for a rational language that is able to know and represent the world and all its aspects perfectly and accurately. Its totalitarian dimension, for Derrida at least, lies primarily in its tendency to marginalize or dismiss all that does not neatly comply with its particular linguistic representations, a tendency that, throughout history, has all too frequently been manifested in the form of authoritarian institutions. Thus logocentrism has, in its search for the truth of absolute representation, subsumed difference and oppressed that which it designates as its alien ‘other’. For Derrida, western civilization has been built upon such a systematic assault on alien cultures and ways of life, typically in the name of reason and progress.

In response to logocentrism, deconstruction posits the idea that the mechanism by which this process of marginalization and the ordering of truth occurs is through establishing systems of binary opposition. Oppositional linguistic dualisms, such as rational/irrational, culture/nature and good/bad are not, however, construed as equal partners as they are in, say, the semiological structuralism of Saussure. Rather, they exist, for Derrida, in a series of hierarchical relationships with the first term normally occupying a superior position. Derrida defines the relationship between such oppositional terms using the neologism différance.

This refers to the realization that in any statement, oppositional terms differ from each other (for instance, the difference between rationality and irrationality is constructed through oppositional usage), and at the same time, a hierarchical relationship is maintained by the deference of one term to the other (in the positing of rationality over irrationality, for instance). It is this latter point which is perhaps the key to understanding Derrida’s approach to deconstruction.For the fact at any given time one term must defer to its oppositional ‘other’, means that the two terms are constantly in a state of interdependence. The presence of one is dependent upon the absence or ‘absentpresence’ of the ‘other’, such as in the case of good and evil, whereby to understand the nature of one, we must constantly relate it to the absent term in order to grasp its meaning.

That is, to do good, we must understand that our act is not evil, for without that comparison the term becomes meaningless. Put simply, deconstruction represents an attempt to demonstrate the absent-presence of this oppositional ‘other’, to show that what we say or write is in itself not expressive simply of what is present, but also of what is absent. Thus, deconstruction seeks to reveal the interdependence of apparently dichotomous terms and their meanings relative to their textual context; that is, within the linguistic power relations which structure dichotomous terms hierarchically.

In Derrida’s own words, a deconstructive reading “must always aim at a certain relationship, unperceived by the writer, between what he commands and what he does not command of the patterns of a language that he uses. …[It] attempts to make the not-seen accessible to sight.” Meaning, then, is never fixed or stable, whatever the intention of the author of a text. For Derrida, language is a system of relations that are dynamic, in that all meanings we ascribe to the world are dependent not only on what we believe to be present but also on what is absent. Thus, any act of interpretation must refer not only to what the author of a text intends, but also to what is absent from his or her intention.

This insight leads, once again, to Derrida’s further rejection of the idea of the definitive authority of the intentional agent or subject. The subject is decentred; it is conceived as the outcome of relations of différance. As author of its own biography, the subject thus becomes the ideological fiction of modernity and its logocentric philosophy, one that depends upon the formation of hierarchical dualisms, which repress and deny the presence of the absent ‘other’. No meaning can, therefore, even be definitive, but is merely an outcome of a particular interpretation.



Q. According to the passage, Derrida believes that:


  1. Reality can be construed only through the use of rational analysis

  2. Language limits our construction of reality.

  3. A universal language will facilitate a common understanding of reality.

  4. We need to uncover the hidden meaning in a system of relations expressed by language.


Ans . D



Q. To Derrida, ‘logocentrism’ does not imply:


  1. A totalitarian impulse.

  2. A domain of representative language

  3. Interdependence of the meanings of dichotomous terms.

  4. A strategy that seeks to suppress hidden meanings in a text.


Ans . C



Q. According to the passage, Derrida believes that the system of binary opposition


  1. represents a prioritization or hierarchy.

  2. reconciles contradictions and dualities.

  3. weakens the process of marginalization and ordering of truth.

  4. deconstructs reality.


Ans . A



Q. Derrida rejects the idea of ‘definitive authority of the subject’ because


  1. interpretation of the text may not make the unseen visible.

  2. the meaning of the text is based on binary opposites.

  3. the implicit power relationship is often ignored

  4. any act of interpretation must refer to what the author intends.


Ans . A